Вы находитесь на странице: 1из 68
[Note:The material ie copyrightod, Al rights reserved. Pdward Goljan, M.D. 2003 PATHOLOGY QUESTIONS [Notes This material is copyrighted. All rights reserved. (Edward F, Goljan, M.D.) 2002 HIGH YIELD STUDY QUESTIONS IN PATHOLOGY ‘Optional: General principles in laboratory medicine questions: this material is covered in the Pathology High Yield section 1. Assuming the use of2 standard devistions to establish the reference interval ofa test, jn atest with ‘reference interval af 10-30 mgfAL, 1 standard deviation would equal... A, 2S B50 re D. 100 E200 Answer=B- mean ofthe tet is 20 mg/dL, 2 SD = 10 sig, therefore 1 SD = 5 mil. 2, Ifthe prostate specific antigen (PSA) tet for prostate enncer is lowered from a refesence interval of Sap 4p is vl ‘increase the number of false negatives J. desman Danser of et patios C.inerease the test's specificity D. — inerease the PV" , _inerease the PV" “Answier: D- increasing sensitivity (less FNs) by lowering the reference interval automatically ineteases ‘the predictive valve of a nopative test result 3. Study the following schematic involving 2 contol group and disease X. Normal Disease X CMS» Reference interval ‘Which ofthe following correctly describes test results inthe space ooeupied by each of the lettered groups? A, Group A: true negatives + false negatives Group B: trae negatives + false positives © Group C: tne positives + false positives ‘D._ Group D: tie positives + flee negatives Answer: group A= Al TNS, pon B= TNS 4 INS, poup C—Hs, 1s, group Dall TPs A pregnant woman in her fret trimester bas an elevated serum T, and normal TSH. Physical exam isnormal. Which of the following best explains the thyroid function stndy results? A. Thyroid binding globulin is increased B. Free T, hormone levels are inereased ©. Estrogen increased the eynthesis of thyroid hormone D. Progesterone increased the synthesis of thyroid binding globulin E. _ Patient has Grave's disease ‘Answer: Aestrogen increases the synthesis of thyroid binding globulin. This automatically increases the ‘otal T, level, since TEG has 4 bound to it. However, the fee T level remains unchanged. A similar question could be an increase in serum cortisol in a pregnant woman who has no signs of ‘Cushing's syndrome, Estrogen also inereases the synthesis of transcortn, the binding. protein for cortisol, hence the increase in serum cortisol, but no sigas of hypercortsolism since fee cortisol levels are normal "Nate: This material is copyrighted. All rightsreserved. (Edward F. Goljan, MD?) 2002, Cell Injury questions |, Hypesplasia is primarily operative in which of the following growth alterations? ‘Appearance of tie affected kidaiey in renovascular hypertension “Thickened bladder wall in a patient with urethral obstruction ‘Barret’ esophagus in a patient with gastroesophageal reflux ‘Enlarged lel atrium in a patient with severe mitral stenosis Galactorshoa in «woman with a prolactinoma Answer: 5~ endocrine stimulation of target tissues i invariably hyperplasia. Choice A is atrophy, choice ‘Bis hyperitophy, choice C is metaplasia, choice D is hypertrophy 2. Which ofthe following disorders is an example of coagulation necrosis? A. Lobar pmeumonia in an aleabolic B, Hepatic sbserss in patient with amebiasis ©. Psendamemranous colitis ina patienton ampicilin D. E Ppa eP Diminished bran mass ina patient with Alzhoimer's disease bolus to the superior mesentcrio artery leading to bowel infarction “Answer: E~ hemorthagio infarction. Choices A, B and C ar liquefuotive necrosis. Choive Dis atrophy, 5, In which ofthe following diseases would you expecta low arterial PO, and. low oxygen saturation (S07 A. Cafbon monoxide poisoning Bron deficiency anemia C. Decreased eartine output D. Respiratory acidosis 5B. Cyanide poisoning Answer: D~choice A has a low 8202, choices B, C, and E have a normal PO, and Sas, choice 4.” Whigh ofthe following disorders is an exampie of metaplasia? ‘A. Increased goblet colls in the mainstem bronchus of« smoker B. Squamous epithelium in the mainstem bronchus of: smoker ©. Proliferative endometrial glands in @ woman on unopposed estrogen 1D. _Hypetkeratosis of the skin ina patient with psoriasis, E._ Mulinucleated giant cells ina granulom “Ansorer: Bobet cells in the mainstem bronchus are an example of hyperplasia. If they were in the ‘terminal bronchiole, it would be metaplasia. Choices C and D are hyperplasia, Choice E represents ‘granulomatous inflammation. Inflammation questions 1. A-d-yearold child with recurrent Staphylococcus aureus infections and an absent respiratory burst ‘MOST LICELY has an. AL defect in spectrin in the eel membrase B. defect in microtubule polymerization C. deficiency of lgG gamma globulins D. deficieney of NADPH oxidase E, _ deficiency of myeloperaxidase ‘Answer: D- patient with MPO deficiency would have a respiratory burst but would still have a ‘mierobicidal problem. Tn addition, there are no azarophilic granules inthe cytoplasm of neutrophils 2 “Wish ofthe following is MOST responsible fr he anittummatry ati of coolers Tncreased leukocyte adhesion to endothelial cells 3. itn of pompbellpne Ay C. Destruction of eosinophils D. Inhibition of cyclooxygenase [Note This material is opysghted, Al rights reserve, (BawardF. Golan, M.D) 2002 E, Inhibition of lipoxygenase Answer: B~ inhibition of PLA, leads toa reduction in synthesis ofboth prostaglandins and Teukotienes. Home 3-5 A. Celus B. Supportive inflammation ©. Fibrinous inflammation D. Fseudomembranous inflamaion Granulomatous inflammation 3. A Staycarcold man with chronic renal fate has chest pain nd scratchy 3 component sound heard over the anterior chest Answer: C~ this is a pericardial frietionrrub 4A febrile 8yeat-old chil, who hes not reeeived any immunizations, bas giayish-hite exudate in ‘he oropharynx nid prominent cervical Answer: D~note, Cdifilepsedemenibeanous colitis has at of similarities with diheria 5. A Rbrile3-yea-old child has a diffse, raised, botre area of inflammation extending around the ‘ight periorbital Gssuo leading to swelling and closure of the eye Answers A~ cellulitis usally de to group A sre, since hyaluronidess (spreading ficir) i produced and allows the infection to spread though the mbeutsncous sao. Staphylococcus aureus produces ‘coagulase, which changes fibrinogen into Fibrin and Jcalizes the infection leading to an abscess with drainage to the surface trough a sinus. 6. A-newbom child has flue of separation of the umbilical cord. Histologic satons of the ‘urpcally removed cord reveal an absence of neutrophil margivation and emigration into the ‘atertial tissu. The ebnial and histologic findings in this case are MOST CLOSELY associated with det in. ‘A. activation ofthe complement aystom 1B. microtubule polymerization the respiratory burst mechanism 1D. anadhesion molecule in neutrophils -E, _ the production of myeloperoxidase Answer: D~ without adhesion molecules, neutophils cannot adhere to the endothelial cells in the ‘venules, hance they eanrot emigrato into tissue. For the umbilical cord to fall off, t must have ‘eutropilinfilteation ofthe issue leading to necrosis ems 7-9 A. Neutrophils B. Mncrophages © Eosinophils D. Masteets F. Plasms cells 7. They represent the epithelioid cells and multinucleated giant cells in a granufomt Answer: B- they are activated by y-interferon released from helper T cells 8 CBaand Cu activate these cells Ansqrer: D~ causes them to release preformed chemicals like histamine and serotonin 9, The granules of these cells contain refiactile material that form Chareot-Leyden erysals in the sputum of asthmatios Answer: C~ recall that eosinophils have erystlline material in their red granules Fluids and hemodynamics questions: 1. Which of the following characterizes early endotoxic (septic) shock rather than hypovolemic or cardiogenic shock? ‘A. Warm skin ‘Note: This meteral is copyrighted. AM rights reserved. (Edward F. Goljan, M.D.) 2002 B. Decreased cardiae output €. _Inereased total penpheral resistance D. Decteaved venous xetur tothe heart Answer: A~ due to the deerease in TPR from vasodilatation (histamine, bradykinin, nitric oxide, release ‘of axaphylatoxins) ‘Which of the following edems conditions represeats a transudate secondary to an decrease in ‘encotie pressure? A. Patient with pneumonia who has a pleural effusion B. Patient with cirshosis who has dependent pitting edema, ©. Patient with edema ofthe arm post-modified radical mastectomy D, Patient with a pulmonary infarction who has 2 left pleural effusion, E. Patient with congestive heart failure with bilateral pleural effusions. ‘Answer: B- in cithosis, there are 2 alterations in Starling’s forces an increase in hydrostatic pressure ‘froin poridl vein Hypettension and a decrease in synthesis of albumin, which decreases oncotie pressure. Choices A and D are examples of exudates. Choice C'is lymphedema. Choice B is an ;ncrease in bydrostatie pressure causing atransudate ‘i eating a patient with sightsided heart failure who has dependent pitting edema, which of the ‘following would be the MOST APPROPRIATE management of the patient's sodium and water ‘tale? ‘Sodium intake — Water intake AL Nochange Decrease Increase Increase ©. Decrease No change D. Decrease Decrease E. _Nochange No change Ansiver: D- patients with heart failure and pitting edema have an excess of slt-and water, hence these ‘must be restricted. 4. A S8.yeacold man with a small cell carcinoma of the lung presents with mental status abnormalities. A CT sean of the brain reveals cerebral edema but no space occupying lesions, ‘Serum electrolytes exhibit a serum sodium of 110 mEq/L. (136-145 mEq/L). There is no evidence ‘of pitting edema or volume depletion. Which of the following is the BEST non-pharmacologie treatment ofthis patent? Sodium intake Water intake A. Decrease Decrease Bo Increase Inceease © Decrease Noschange D. — Nochenge Decrease E. Nochange No change ‘Answer: D-~ the patient has inappropriate ADH syndrome. Since ADE realpsorbs solute fice water feom the Kidneys, the excess water distributes in both the ECF and ICP compartments. Dilutional Iyponatremia favors an osmotic gradient moving water info the ICF compartment, The Rx of choice isto restrict water, sine the TNs is normal 5, A @2-year-old man has & ruptured abdominal aortic aneurysm. with retroperitoneal hemorrhage. ‘Which of the fllowing findings BEST represents this patient's clinical disorder? LVEDP) TER Cardiac output ‘A. Normal Normal Normal TB. Tnoreasod_ Increased Deoreased C. Decreased Increased Decreased D. Deeeased Decreased Decreased LVEDP left ventricular end-diastolic pressure), TPR total peripheral arteriolar recistancs) 4 ‘Note: This material is copyrighted. All rights reserved, (Raward F. Goljen, M.D.) 2002 Answer: C- the patient isin hypovolemic shock, hence the cardia output is decreased, TPR is ineteased ‘due fo catecholamines, vasopressin, and AT I, and the LVEDP is decressed, sie plasma volume is decreased. LYEDY TPR Cardiac output Interpretation A, Normal Normal ‘Normal ‘Normal B. — Inofeased. .nereased Decreased Left-sided hese failure D. Decreased Decreased Decreased Septic shook 6. Which ofthe following isa coagulation factor that i utilized in both an occlusive venous thrombus sadn an arterial rombus? A. Factor XI B. Factor Vil Factor DX Di FabtorX E. _Pibrinogen “Answer: E~ fibcioge is preset in both venous thrombi (Sibrin lot txpping RBCs, platslets, WHCS) anid in areal thrombi (aggregated platelets held together by fibrin). 7. 429 yr. old man sustains bilateral femoral bone factures and mile pelvic fractures from 2 ‘mountain bike accident. Forty eight hours ater, he develops a sudden onset of dyspnca, petecial lesion, and mena status allatons. Laboratory stuios reveal hypoxemia and Uronbocytopens “The mechanism for this patent’ eincal disorder is most closely associated with. A. disseminated intavascsiar coagulation B, pulmonary embotism © fatembotization D._sirembolization E. _ pucumonia " C-the patent ha ft embolization, Note the time delay before symptoms occur, Nuttition questions: optional. Material in High Yield 1. A.25-year-old woman has not had her period for the last 8 months. She is 52" and weighs 90 pounds. A urn pregoaney test is newaive, She states that she bts been trying to lose weight for her ‘upcoming wedding. You order s batty of tess and give the patient an intramusenlar injection of progesterone. Ten days later the patient returns t0 your office and reports that she fad no ‘withdrawal bleeding. Laboratory tests reveal the following: serum prolactin is normal, serum FSH and LH are low, serum TSB is normal, serum estradiol is low, and serum cortisol and growth hormone are inereased, Based on thee findings, you strongly suspect the patient has. A. primary ovarian disease oy ye ‘Answer: D= anorexia nervosa. Amenocthea leaves the patient estrogen deficient and prone to seporn S hul paced on bt conte! ils. Nos how theses amon (ris, 2 Follicular hyperkeratosis, night blindness, anda hemorrhagic dathesis are expected in patient. ‘with cystic fibrosis with scurvy with hypothyroidism who is pure veran ‘who is bulimic Fooee [Notes This material is copyrighted. All rights reserved, (Edward F, Goljan, M.D.) 2002 Answer: A-vitamin A deficenoy account forthe ist 2 symptoms, while vitamin K deficiency acoounts ‘or the hemortbagic siaheis. Patents with CF have mlabonption and las all te ft soluble vitamins 3. Which ofthe sigs or symptoms characterize ft soluble rather than water soluble vitamin deiceney? ‘A. Perfollicuar hemorrhage B, Bone puinand ©. Peripheral. D. _Ophthaimoplegi, confision, and tax E,_ Hyperpigmentalion in sumexposed areas ‘Answer: B- vitamin D deficeney causes bone pain an tetany, since the bone snot mineralized property nds tof (osteqnalaca) leaving ieprone wo Facies, Typocaleea explain the tetany. Choice A is eoury, choice C is pyidexine deficiency, choice Dis thamine deficiency, cboiee is pellag, For niin daiieny. 4. Which ofthe ign or symptoms are mare prominently found in marasmus than kwashorkor? AL Hlpatomegaly Biking edema G. _ Redused oil Iymphoeyte count 1D, Flaky point dermatitis BE. _ Broomstick extremes Answer: E-all the ober Bdings characterize kwashiorkor, where there f anormal total aloe intake Fellag ll MOST-LISEL ent. 4. Pellgra will MOST LIKELY develop ina pate. ‘A. who is takiigioniard 1B. whose diet primarily consists of com, C. whoisspure vegan D. hosting nots ac to lower pds E _ with maldigestion secondary to chrane pores Answer: B com eotains niain which is in a bound form that cannot be absorbed. Chace A is Prridonine deficiency, choice Cs By deficiency, choice D isa by roductof niacin, choice B- Inildigestion will aus fa soluble vismin defciensies A, DF, K. 6 Which ofthe following more often asocetod with anorexia nerwosa than bulimia nervosa? A. Hypotalemia BL Metabolic allaosis ©. Normal body image 1D. Normal serum gonadotropins E._ Osteoporosis Answer: F~ diet lack of estrogen. All the ates findings at nore commonly sen in bliin. 7. Which of the following laboratory tat abnormalies would you MOST expect it paticnt with roid obesity? ‘AL Tnoreased serum TSHL BInereaed 2¢-hr urine for ie cartisol C. _Tneremed fasting glucose 1D. Inoreaed 24-hr urine for 17-etoteroids Barend serum DEA -elfe Answer: C~ increased adipose down-regulates insulin receptor synthesis predisposing the patent to type DM or ghveose intolerance. A urine for fee cortisol clearly separsten obesity from Cushing's symdrome the ltr having en ireas in ree levels in the urine & Which ofthe following vitamin deficiencies would you expect in child maintained on unfortfied goats milk? A. Ascorbic acid ‘Note: This material is copyrighted. All ights reserved. (Edward F. Goljait, M.D.) 2002 B. Thiamine © Niacin D. Riboflavin FE, Folate Answer: B~ goat’ milk is deficient in folate and pyridoxine 9, Which ofthe following vitamins would be deficient in a newborn child with anemia: whose mother fsa pore vegsn? A. Ascorbic acid B Thiamine C. Niaoin D. Pyridoxine ER Bo ‘Answer: I~ pure vegans do not eat meat or dairy poduns, hence they arc auscentible to By deficieney. Vegas who are pregnant should be placed on Bs; supplements in addition to the usual prenatal vilamins containing folate ad iron, 10. A.65-year-old woman complains of bleeding gums after brushing her fecth, easy bruising, and pain “in ber logs when walking: Her platelet count is normal, The pathogenesis of her disease is Mi (CLOSELY related to... A adeficency of ATP B. aofactor deficiency in collagenase lack oflhycronyiation of lysine and proline D. _ncofacter deficiency in iysyl oxidase E, _ platelet dysfunction the pationt has soury. Vitamin C hydroxylates lysine and proline. This is the binding site for the orss-bridge that strengthen collegen by eteating a triple helix. Hence, in seurvy, collagen is weakened. 11. A.30 year-old tan develops an acute onset Of eonfson, ataxia, nystagmus, and ophthalmoplegia ‘shortly after the administration of an intravenous solution containing 5% glucose snd normal saline. ‘The pathogenesis ofthis patient's neurologic disorder is most closely relate to. A. central pontine myslnolysis 2B. hamine defcieny ‘Purkinje ell atrophy 1D. viralencephalits E, _ Badeficiency ‘Answer: B- the symptom complex describes acute Wernickes encephalopathy. Aleabol exces is the ‘MCC of thiamine deficiency. Thiamine is « Cofactor forthe pyruvate to acetyl CoA. reaction using setydcogenace, Giving the patient glucose in an TV eausos the rst ofthe thiamine to bo ‘used up, hence precipitating acute Wernicke's encephalopathy. Always give TV thiamine before hanging up uvose. Optional Genetics questions: material is covered in High Yield Anew 1. ASl-yearold woman delivers a full-term baby thet has repeated vomiting of bile stained materia. “A flat plate of the abdomen reveals air im the stomach and proximal duodenum and no aie in the remainder of the bowel. The matemal serum c-fetoprotein level is low. Tho baby has 46 chromosomes. The mechanism ofthe childs disease is most closely associated with ‘a Mendelian diserder ‘a Robertsonian translocation ‘ondisjunotion in meiosis 2 point mation of a nucleotide ‘amicrodeletion dicorder Room ‘Note: This material i copyrighted. All rights reserved. (Eéward F. Goljen, M.D.) 2002 “Auswrer: B~ the paticat has Down syndrome based on the low serum a-fetoproten and sign of dodenal ‘aresia, 46 chromosomes indicates a Roberlsonian translocation, where the mother had 5 ‘sbromosome, where the 2 chromosome 21s ate fused info | chromosome. The chromosome from ‘the mother essentially has 2 chromosome 21s + the 1 chromosome from daddy Jeads to 46 chromosomes, however, there ae 3 fanctional chromosome 21s. Nondisiunetion is the eause of ‘eisomy 21, 2, Iban Affican American woman with sickle cell disease has children with a man lacking the ‘bnormal Brohain, you would expect. A. 25% oftheir children to have sickle ool tat 1B. 50% of theichldcen to have sickle ell trait ©. 25% of ther children to have sickle cel disease D. 50% of ther children to have sickle eel disease E, _alloftheicehildten to have sickle cell ait ‘Answer: since the woman is homozygous for the sickle gene and the man is nora, al he children ‘will have side trait 3. A 17-year-old adolescent presents with primary amnorthea. Physical exam reveals normal ‘secondary female sex cheracterstics. Discrete masses ate noted in both. inguinal amas, A ‘pes en of vgn indeste alin posk You would eps hi pet tao re. svprostate gland seminal vesicles ‘au androgen receptor deficiency ‘one Barr body on'a buceal smear ‘ovaries in the inguinal canal, ‘Answer: C- testicular feminization, ‘This is & SXR.(nsle) disorder with absent androgen receptors. ‘Therefore, fetal testosterone cannot stimulate the development of an epididymis, seminal vesicles, ‘or vis deferens from wolffan duct structures. Similarly, dThydrotestasterane cannot fuse the Tobia into a scrotum nod extend the olitris into. penis and carmot lead to formation of a prostate gland. ‘eft unstimulated by DAT, the external genitalia reamia female in appearance, The lower 2/4 of ‘the vagina can develop, since it represents the urogenital sins. All mollerian structures are \estroyed by apoptosis, so pstients do not have tubes, ufos, cervix, othe upper one-third of the ‘vagina, hence the blind pouch. 4, While examining 13-year-old boy during a routine physicel examination, you note bilaterally ‘enlarged, non-tender testicles that do not transilumimae, a high arched palate, and a mid-aystolic ‘jestion click followed by a short murmmr, You call the sehool counselor an fd that the chil bes ‘a moderately severe attention deficit syndeome. Which of the following studies would you recommend on this boy that would best explain all of the sbnormalites noted on the examination? A. Echocardiogram B.Baoeal sear ©, Secum gonadotropins D. Wdentifeation of tiplet repeat E, Chromosome study on his father ‘Answer: D~ patent has fragile X syndrome. This isa triplet repeat disorder, hence the discase gets worse in fue generations in both the affected males and the female carriers. Some geneticists, Cerefore ‘all this a sextinked dominant condition, since female carriers may express the disease. However, ‘thers consider it sexlinked recessive disorder ODE 5. Prader-Willi and Angelmian's syndronit have different clinical features, however they both share a ‘dofect atthe same location on the same chromosome. This is an example of... A. variable expressivity B. a Robertsonian translocation genetic heterogeneity D. genomic imprinting, ‘Note: This material is copyrighted. Al rights reserved. (Rdward F. Goljan, MD.) 2002 E, abalmced translocation “Answer: D- microdeletion syndrome on chromosome 15. Ifthe chromosome was maternally decived, ‘the patient develops Angelman's syndrome (happy puppet" syndrome). IF the chromosome is paternally derived, the patient develops Prader Willi syndrome. 6. Ifthe carier rate forthe siekle cell abnormality is I in 12, the prevalenve of sickle cell disease is approximately 1 in... A Ms B28 C576 D. 720 E1440 Answer: C: Sickle cell Answers: B, D, E, G- they have Pb poisoning with peripheral neuropathy and the abdominal coli. The a ‘int acd i pottery often contains Pb. Other scensros-used on boards snelude eating old paint, “woking in a hutomobie factory Generate batteries), drinking moonshine lead lined radistos) A33 year-old Afican-American modical missionary, wh teeemly returned ftom a 2 year Your in ‘India, is diagnosed with leprosy. Aer | weck af therapy, he develops fever, chills, low back pan, and dark colored urine. A CBC reveals a heooglabin of 6 g/l (13.5-17.5 pa), leukocyte coun ‘ 15,00am? (4500-1 1,000/mm’), and a platelet count of 450,000/mm? (130,000-400,000/mm), ‘A corrected reticulocyte count i 18% (0.5-1.5%). A direct and indirect Coombs testis nepative ‘The peripheral smear exits polyehromacia and RCs missing parts of hele membrane. There is positive urine dipstick for blood. The urine sediment is reported as normal. Which of tho following -Ratements apply to this case? SELECT 3 ‘A. Hinz bodies are likely present B. Predominantly extravescular hemolysis © Low RRC glutathione levela D. Ration i taking dapsone 1B. Autosomal recessive disease swers: A, C, D- patient has G6PD deficiency (SXR disease) with hemolysin secondary to dapsone. "During acute hemolysis enzyme assays are asually normal, since only the cells containing the enzyme remain behind, wile these witout the evzyine are bemolyed. 10, An afebrile 65-year-old man presents with fatigue and substeral chest pain with exertion that fs as ‘lieved by resting. Physical exam reveals a blood pressure of 100/70 mm Hg, pulse of 110 ‘beats/inute (Gimitished amplitude), pule conjuictiva, and a arch grade I-IV systolie murmur with radiation into che carotid arteries. The intensity ofthe murmur increases with expiration and ‘when the patient is lying down. A CBC exhibits a moderately severe ancmix with «meat ‘corpuscular volumne (MCV) of 76 jum’ (80-100 jum’), hemoglobin of 7.5 giAL (13:5-17.5 g/dL), Teakoeyte count of §000/mmn’ (4500-11,000/ma"), and a platelet count of S00,000/um* ((150,000-400,000/mm') The peripheral smear uncovers numerous fregmented RBCs, The corrected reticulocyte comt is 15% (0.5-1.5%), There isa positive urine dipstick for blood but [RBCs are not present in the sediment. Which ofthe following apply to this ease? SELECT 4 ‘AL Positive diect Coombs test B. Low plasina haptoglobin C.Schistooytes 1D. Hemoglobinaria E, Attic regurgitation F Low serum ferritin saswers: B, C, D, F= the patient has aortic stenosis and x macroangiopalbie intravascular hemolytic ‘anemia with schistooytes. Loss of Hg in the urine has caused iron deficiency. Thrombocytosis is ‘common in chronie iron deficiency. B [Note: This material is copyrighted. All rights réserved. (Edwatd F. Golan, M.D.) 2002 11, An asymptomatic, normotensive 21-year-old Aftican-American woman is noted 10 have imieroscopic hematuria on an otherwise normal physical exam. The urine eultare returns negative. ‘Arenal ultrasound is reported as normal. Her hemoglobin concentration is 12.5 g/L. (12.0-160 AL) aud the peripheral smear is reported as normal, Based on these Findings, which of the following isthe next best step in the management of this patient? A. Siekle cell preparation 1B. Reticuloeyte count © Renal biopsy D. Cystoscopy E.Ne further work-up ‘Answer: A~the patient has sickle cell ait with milcroinfarotions inthe renal medulla. 0, tension is low ‘enough in the medulla to induce sielding. 12, A pregnant 2i-yearld African-American wornin has a mild microeytic anemia with = norms! A & 2 3B. _inoreased Fgb Ht ‘Answers: B, C, D= the patient has Bthalasersia minor, Since Phan synthesis is dcrewsed, c-chins can combine with 8cains to produce Higby and eomibine with trehain ia produce gbF. HgbA is desreased due to the lack of P-chains. 13, A-dé-yearold man with diastolic hypertension develops fever, jaundice, and 2 severe hemolytic fnemia. while taking methyldopa, ‘The hematoerit is 15% (41~$3%) and the uncorrected Teticulosyte count is 24%, Spherocytes and polyehromasia are present in the péxpheral smear. ‘Which ofthe following most ikely apply to this ease? SELECT 3 ‘A. Intravascular hemolytic anemia B. Reticlooyte index is 4% CC. Unconjugated hyperbilirubinemin D. ‘Type hypersensitivity reaction E, Autoantibodies against Rh antigens Answers: B, C, F-autoimmune hemolytic anemia, Must correct forthe anemia andthe polychromasia. ‘Typo Ithypersensitvty reaction. 14, A'65-year-ld man presents with fever and chills. Physical examination reveals generalized, non- tender lymphadenopathy, bepstosplenomegaly, and scatiréd petectia and ecchymoses over the ‘nferorchest.A CHC tepor indicates a hemoglobin of 8.2 yA (135-1755 gl, leukooyte count ‘of 70,000/mm’ (4,500—11,000/mm*) with 90% mature and fmmature appearing lymphocytes, and x Platelet count of 80,000/nm? (150:000-400,000/ean?). The peripheral smear contains murserous ‘smudge cells, A blood culture is positive for Sireplocnccus pneumoniae. The total serum protein concentzation is 4.0 g/dL (6.0~7.8 g/dL). A secum protein electrophoresis exhibits fla y-globulin reakcend decrease albumin. Which of the following apply'to this case? SELECT 3 “Leukemoid reaction secondary to sopsis Hypogatmaglobulnemia ‘Chron Iymphooytic leukemia Low leukocyte alkaline phosphatase seare B cell malignancy ‘Answers: B, C, E— the patient has CLL with hypogammaglobulinemia. Tis the MC leukemia and cause ‘of generalized lymphadenopathy ater 60 ys of age ROOR > “ ‘Note: This material is copyeighted. Al rights reserved. (Edward F. Goljan, M.D.) 2002 15. ‘A Axyear-old boy presents with fever, epistaxis, and testicular pain. Physical exam reveals ‘generalized, non-tender lymphadenopathy, hepatosplenomegaly, sternal tendemess to percussion, fad widespread potechia and eochymoses, Both testicles are enlarged, slightly tender, and do not transilluminste, The CBC report indicates a hemoglobin of 6 g/L (11.0-140 g/dL), Teakoeyte count of 30,000/mm? (5,500-15,500/mm), and a platelet count of 50,000/mm? {(150,000-400,000/mz?). A bone manrow aspirate reveals sheets of cells similar to those present in the peripheral blood. Which ofthe following apply to this case? SELECT3, ‘A. Cells are most likely CALLA positive B. Celle are most likely Tat positive C. Acute lymphoblastic leukemia, D. Lymphoid leukemoid reaction B._Leakernia derives from trilineage myeloid stem ea “Answers: A, B,C: the patient has acute lymphoblastic feukemia 16. ‘A aSyearold man presen wi fever, weight los, and sweating. Physio! exam reveals = ‘normotensive individual with gonerlid, nontender Iymphadenopathy and massive hhepatosplenomegaly. The CBC report indicates a hemoglobin concentration of 7 pi. (150,00-400,000/m"), iekoeyte count of 150,000/inm? (4,500-11,000/mm’), and a platelet cet of 650,000? (150,000—400,000/nan’) ‘The peripheral sma emonstrtes neutrophils at fl stages of development (19% myeloblass), mature and immature eosinophils and basophils, and thrombocytosis with giant plstceis. A bone marow aspirate is dy, however, the bone marrow ‘biopsy roveals« hypercllular marcow with an increase in reticulin fibers. prints of the biopsy reveal a similar differential count as that observed inthe pecipheral blood. Which ofthe following {ional Inboratry test abuormalites would you expect i his patent? SELECY 3 Tow leukocyte alain phosphatase score Positive truate resistant acid phosphatase tain “Avec rods in myeloblasts Positive Piladciphia chromosome study Positive br fusion gene sty {8:4 translocation Fmuom> Answers: A, D, E— chronic myelogenous leukemia, Blasts do not contain Auer rods (Only abute 17. ‘myelogenous leukemia has Blasts with Auer rods) A 19-year-old African American woman presen's with fatigue and exercise intolerance. She has 2 history of menorrhagia and spocadically takes ferrous sulfate tablets. A. CBC reveals mild nommocytie anemia, a low normal WBC count, a normal platelet count, and normal RBC ‘morphology. A corrected reticulocyte count is <2%. The next most'important step isa. ‘A. serum feritin| 1B Coombs' test C. serum folate/Bp BH lento Answer: A~ early stage of iron deficienoy. Sickle cell trait doce not have aneena, Recall tat all the iron 18. studies (iran, TIBC, % saturation, ferritin) are abnormal before there is any anemia, The anemia is first nonmooytic and then becomes microcytic ‘An afebrile 80-year-old man with the myelodysplastic syndrome is symptomatic with a 7 gif. hemoglobin, He is given 3 units of packed RBCs and on the following day has a hemoglobin of & ‘end. His direot Coombs’ tost is negative. A dipstick of rine for blood is negative. The MOST LIKELY cause for only a1 gun rise inthe posttransfusion hemoglobin concentration is... ‘A. destruction ofthe RBCS in the bone marrow 1 amicroangiopathic hemolytic anemia C._anautoimmune hemolytic anemia 1D. destruction ofthe RBCs in spleen. (ote: This material is copyrighted, All rights reserved. (dward F. Goljan, MD.) 2002 Ea gastrointestinal! bleed Answer: B~ think most common~ lack of reponse to transfusion is mast commonly due to GT bleed. “The Hab should increase by 1 gv. for every nit of packed RBCs andthe Het should increase by *%. 19, A 2yearold college student presents with petechia, eochymases, epistaxis, generalized ‘ymptadenopathy, end hepatosplononepily. A CHC reveals @ normocytic enemia, @ total WBC count of 30,000 cells, and thrombocytopenia, Abnormal "bast cells” are noted inthe peripheral sacar. The PT and PTT are prolonged, and the D-dimer testis positive. Which ofthe following apply to this ease? SELECT 2 ‘A. Positive tartrate resistant avid phosphatase stain 1B. Therapeutic response to trmsetinoic acid . Low leukocyte allatine phosphatase score D. (5:17) tnslovition 2, _nvasion ofthe gums Answers 8, D- patient has acute progranulosytic leukemia with DIC, A positive TRAP stain is bairy ‘eel leukemia snd invasion of the gums is acute monocytic lcukemia. A Tow LAP is chronic ‘granulocytic leukemia, 20. Which of the following is more often associated with F-thalassemia minor than iton defiiency? SELECT? ‘A. LowMcy B. Inereased RDW ©. Tacease serum fevtn D. Normal to hh RBC count E, Abnormal High electrophoresis Answers: DE temember that ron studies are normal in mild thalassemia («or ). creased RDW is characteristic of iron deickeny, Both have alow MCV. 21, 4 28yearold woman present with fever fatigue, and scleral jters, She is uot taking any presripion or overthe-comter medications. Physical exam reveals yeneraizedpeiafil Tymphadenopathy, hepatosplenomegaly, and a facial rach in a buterfly distbution, A CBC exhibits a Hpb of 6 gL, a slighty ineeased MCV, thrombocytopenia, and slightly Tow WBC count. The peripheral smear demonstrates spherocyies, REC polyelrontiais, and oceasionl imlated RCs. No hypersegmented neutrophil are prevent. The uncemevted retculoyte count 15%. Which of the following apply to this case? AL Bilin inthe wine B. Positive dtect Coombs! test C. Unconjugated D. — Reticloayte index < 2% F. _ Positive serum antinosler antibody test Answers: B,C E~ patent wilh SLE and a warm AIHA, which is an extravascular hemolysis leading to sninereae in UCB and jeundice, UCB cannot be filtered in the rine like conjugsted bbrubin The reticulooyte index =4.5% 22. Youwoulé expecta matte RBC to. SELECT 2 A. ile lactate dehydrogenase 1B. lize glucose 6 phosphatase convert glucose into glycogen D. utilize pyruvate dehydrogenase z have anet guin of 2) ATP snd (0) NADH. “Answers: A, B- they have no mitochondria and use anaerobic glycolysis for energy 23. An afebrile patient with SLE bas a mild normocytic anemia with an elevated leukocyte count exhibiting neutrophilic leukoeytasis, lymphopenia, easinopenia, and a normal platelet count, There 16 ‘Note ‘This materi is copyrighied Al ights reserved. (award F. Golan, M.D.) 2002 js wo left shift or toxic granulation present in the smear. He stool guaiae is postive. The findings ewe MOST CONSISTENT with. ‘A. avate leukemia B. aviralinfoction © abacterial infection D. gramacgative sepsis B. _ corticosteroid therapy ‘Answer: E~ decrease adhesion molecule synthesis and destroy lymphocytes and eosinophils . 24. Which of the following hemolytic ancmias are primarily due to extravastular removal of RBCs? Cold autoimmune hemolytic anemia “Answers: B,C; NOTE~ the others ae primarily intravascular hemolysis 25.” Which ofthe folowing canecty dsctibe anemia? SELECT 2 AL Bypoxemia B, Low oxygersaturation Decreased oxygen content D. —Laftshifted oxygen dissociation curve #, Tissue hypoxia Answers: C, E~ only the High concentration is decrease. Gas exchange is normal, s0-the PaO, and $40; should’ be normal. 26. A 65-year-old man presents with joint pains, palpable purpura, aad hepatogplenomegaly. There is no evidence of lymphadenopathy. A CBC exhibits a normoeysc anemia, thrombocytopenia, and neutropenia. A few “blast cell with imegular cytoplasnie borders ae noted in the pecipheral blood. A hone marrow biopsy reveals « monomorphic infiltrate of cells with abundant cytoplasm hhaving a “Sred eas” appearance. A special stan is pending. Which ofthe following, apply to this case? SELECT 2 A, (9322) translocation B, Epstein-Bar virus association CC. Beell malignancy D. Positive tartrate resist acid phosphetas stain E, Low leukocyte alkaline phosphatase score Answers: C, D~ the ptfont has hay cel eukerin 27, An 82-year-old man has’ long history ofa severe slightly macroeytic anemia and pancytopenia. He requires a transfusion of packed RBCS every 2 weeks in order to maintain his Hib condentration ‘round 10 pL, His peripheral smear consistently shoves a dmporphie RBC population associated ‘with oceasional myelobasts and progranulocytcs. A bone marrow aspirate reveals numerous ringed sideroblais and a 10% myeloblast count. This patient MOST LIKELY his. ‘A, aoute myelogenous lenkemin Agnogenie myeloid metaplasia the myelodysplastic syndcome chronie myelogenous lekeria neutrophilic leukemoid reaction Answer: C- these patients frequently progress into en acute myelogenous leukemia. Small and large [RBCs and blasts are common in the peripheral blood. Ringed sideroblasisaze also a commen feature in the bone matrow. B. c D. E ‘Note: This material is copyrighted. Al ights reserved. (Edward F. Goljen, M.D.) 2002 28, _AS3-year-old woman with an abnormal mammogram ofthe left breast, palpable leftsxillary lymph nodes, and low back pain is noted t9 have a ald normocytic anemia, thrombocytosis, slightly slevated WBC. count, and a peripheral smear demodstraing nucleated RBCs, and occasional ‘wopanoojes ad gees, Te ndings a cones: wih SELECT 2 chronic myelogenous leukemia, 5 mokrmychwesou etn €. — mamow infiltrative disease D. _loukoerytheoblasie smear E,_leukemold reaction ‘Answers: C, D laukoeryioblastic smear fiom metastasis to bone. Tumor is pushing normal marrow hematopoietic cells nto the peripheral blood. 28, Eoin eommoly eosin wih SELECT 4 pinwors, Bono aston ©, malar D. amebiasis BAddison’s disease F poison ey G.penielin skin rash H. __strongploidiasis Answers: B, E, G, Hi NOTE: among parasites, only invasive hekminths produce eosinophil. Pinworms donot invade, 30. Texic granulation, Je shi, and entropic leakoeytosis are expected in... SELECT 4 AL acute B Crolafe disease © soute dvertintits D. acute cholesystis 5, aeie myoeardal intaction, F. cheumatoid arthritis Answers: A, C, D, 1 the first 3 are bacterial infections due to £: coll: Catecholamines and tissue necrosis explain neutrophilic leukocytosis i an AMI 31. Answers: 4, C- parvovirus also produces aplastic aneinia in patents with an underlying hemolytic soomih ep ek el ens canna meron), 32, A2erold child has a microcytic anemia, Which ofthe following is the fist step in the work-up of D. Direct Coombs test E,_ Hyb electrophoresis Answer: B- most microeytic anemias in children are due to a bleeding Meckel's diverticulum. 33. A.55-yrold man bas a microcytic anemia, Which ofthe following isthe first step inthe work-up of the patient? A. Serum ferritin BL Stool guaiae 18 "Note: This material is copycighted, Al rights reserve, Hdward F, Golan, M.D) 2002 ©. Bone marrow D. _Dirvot Coombs test E. _ High elestrophores Answers B~ most microcytc anemia in sdults over 50 are due o eoton sancer 34, In lead poisoning, the encephalopathy is directly atibuted to A. mrincrease in REC protoporphyrin 1B, aminoreaso in -aminolevulinic acid ©. Pb depositing i the brein D. amaltertion in Starling’ forces ‘E. _tissue hypoxia secondary to-enemia Answer: B somehow, it ineresses vessel permeability leading to veebrel edema ‘38. "laa pationt with Bre deficiency who i being treated with pharmacologic doses of flat, which of the following will be corrote? ‘A. Megaloblastie anemia B. Glossitis cD 1D. Vibratory sensation inthe lower extremities E__Proprioception ¥. Dementia ‘Answers: A, B, C NOTE: the newologie deficits remain, hence the importance of making the eorrest siagnosis 36, Newboms with sickle cell disease do not have hemolytic or vasooeclnsive orises at bieth sinee.,.SEERCT 2 A. theigh concentration of Hah F inhibits sickling B. levels of Het 8 are not high enough to induce sickling ©. High A inhibits silting 1D, splenic macrophages are of insufficient mamber to remove sickle cells E. the spleen traps the sickle ells Answers: A, B~ Tigh A is never present in sickle vel disease 37. A.19r-old Aftican Americen man with sickle cell disease develops bone pain jn the feinar. A radionuelide bone sean reveals a lytic lesion in the metaphysis ofthe fem. The pathogen most "ly spas forthe patents one rd Staphylococcus aureus Sireplococens pnewmoniae Salmonelia paratyphi Pocuiomonas aeruginosa Henophitesinfiuenzae Answer: C- in children without sickle ool tease, Staphylococcus aureus i the MOC of osteomyelitis, “not Salmonella 38. Ina patient with sickle cell trait, you would expect the MSTII endonuclease studies to seveal. 135 kb segment L1SKb fragment 0.2 kb fragment A“ Q)scgmenss none one B,—(i)segment (1) fragment (2) fragment c one (©) fragment fragments Answer: B: the abnormal chromosome has (1) 135 kb segment, while the nonmal chromosome is cleaved into (1) 1.15 Kb fragment and (1) 0.2 Kb frogment. Potent A.has sickle cell disease, with 2) ‘uncleaved 1.35 kb segments. patient C is normal, and both 1.35 kb segments have been cleave nto (2) 1.15 kb fragments and (2) 0.2 kb fragments BpOEP 19 ‘Note: This materials copyrighted. Al sights resrved, (Edward F, Golan, M.D) 2002 Tymphoproliferative questions Items 1-2 ‘A 65-year-old woman presents wth fatigue and pai in he lower ack ad ibs. She tes hat hor urine flow has also decreased dramatically in the last few days. Physical examination reveals sternal and vertebral percussion tendemess and bilateral conjunctival pallor. A CBC exhibits extensive rouleaux, a hemoglobin of 7.5 g/dL. (12.0-16.0 g/iL}, a total leukocyte count of 4300/mn? (4S00—11,000/mm, and a place count of 125,000? (150,000—400,000/mm2). A urinalysis exhibits a 2+ dipstick for protein and 4+ precipitation using sulfosalicylic acid (SSA). Renal tubular casts are noted in the sediment. A chest x-ray reveals generalized esteopenia in the ribs and vertebra aswel slip Ite lesions in the ibs. 1. Based on the patient's history and preliminary laboratory findings, which of the following addition sbnocaes would you expeat? SELECT 3 AL Bypereaoemia Normal erythrocyte sedimenistion rite teh monoclonal spike in soni DD. lteresoed light chains the urine B Malignant plasma cols ina bone marrow agate “Answers: A, D, Ete patient bas matic myctoms. Note te tre ings that sree for BY peetein (disparity between the dipstick for protein and SSA, the latter detecting both albumin and globulins, Wile the former only detect abun), Tabuler ont inthe urine imply tht acute tubal necrosis dnasaleady eared. 2. ‘Whataddional ests ould bs permed to contin the diagnosis? SELECT 2 ‘Bone marrow asiate Renal bepey Radiomelide bone san Intravenous jyelogram VE) Serumrine mmimocletroporess “Answers: A, Ean IVP would farther exacerbate the patient's renal disease. TEP idetifies the abuormal Ind light chain. ena 7 RopR> Histooytosis X Sezaty emdome Hodgkin's disease rats iymphoma Jnmunoblastic lymphoma ‘Waldenstrom's macroglobutinemi ‘L.A Dear gil presets sith colicky pain secondary to entrapment of small owel by enlarged pareaoric lymph nodes. A section of lymph node removed at laparotamy reveals a diffose neoplastic inflate of srl, round Iymaphocyies with "starry sky” sppearace. Answer: D:B cell malignancy with a (8:14) of te c-myc proto-aneagene 40° GByearold man presents with multiple plaquelike lesions on his skin, generalized Ismphadenopethy, and hepatosplenomepaly. A biopsy reveals atypical Iymphoeytos inating the epidermis, Neoplastic CD, positive lymphocytes with prominent nuclear clefts are noted in his peripheral Blood “Answer: B- Sezary syndrome. Is called myeosisfangoides ifthe cells are notin the blood. 5. A 4-yearold child presents with exophalmos, polyuria, and mullple Ite lesions in the skull. A ‘bone marrow aspirate reveals an infiltrate of neoplastic cells that are CD1 positive. Answer: A-Hand-Schiler-Christian disease, CD1 is histiocyte marker. BEE OR> 20 ‘Note: This mateil is copyrighted. All ight reserved. (Edvard F. Gotjan, M.D.) 2002 6 A @®-yearold man has @ nonmocyic anemia, on clevated exvosyte sedimentation ral fencrlized lymphadenopathy, hepatosplenomegaly,hypervisosity syndrome and an abnormal Finding ona serum protein electrophoresis ‘Answer: F- Waldenstromis macroglobulinemia~ an TgM monoclonal spike must have been present. ‘Miligte myeloma does not usualy have an TeM spike 7. A2Byrold woman has en anterior mediastioal muss and wonder lymphadenopathy in the right supraclavicular node Answer: C: nodular sclerosing Hodghin'sGiteas, This is the classi presentation of nodular scleasing "HD in women or men % Resistive exrdiomyopathy, ameroplowia, and nephrotie syudrome are wll associated wit disorder characterize by. ‘A. neoplastic histioaytes ina bone marrow aspirate. 1B. neoplstie plasma cells a bone marrow spirale CC. seoplatcIymphoytes with at) tsodation D. _Iymphoplasmacyoid cells anda monoclonal increase in IgM 1. _ material exhibiting apple geen birefringence under polarized ight Answer: E: these are ll features of amloidois ‘Hemostasis questions L.A 22-year-old Aftican American woman with no previous bleeding history develops persistent bleeding of her gums and tooth socket following dental surgery. Additional history zeveals problems related fo heavy menses that significantly resolve when she is taking birth control pills ‘nd recur fo the same level of severity when the diseontinues the medication, Hematologic studies xeveal the following; partial thromboplastin time (PT) 55 see (28-40 sec), prothrombin time (PT) 12 see (I-15 seo), factor VIM: coagulant 30% ($0-150%), factor VI: antigen 40% (60~150%), bleeding time 15 nin (2-7 min), platelet count 300,000 mn? (150,000~400,000 min’), hemoglobin 13,0 gill. (12.0-16.0 g/dL). Which of the following Inboratory tests is mast usefbl in confirsing, the cus fer edn? Leukocyte count Bites elec C. Hemoglobin electrophoresis, 1D. Siokle cell preperation E, Serum ferritin ‘Anowar the patient has on Willebrand’ disease ‘A 2ieyear-old man requises a 100t canal for an abisbessed footh for whioh he has been taking pin zedication. On the day prior tothe procedure, he develops a sovere nosebleed, which proms his dentist to onder a few laboratory studies, which areas follows. PTT 35 sec (28-40 sec), PT 13 eee ‘Q11-15 sec), bleeding time 16 min (2-7 min), platelet count 200,000 mn (150,000—400,000 zx), Higb 15.5 gldL (13.5-1755 g/dL). Which of the following hemostasis abnormalities is most kely ‘preseat in this patient? ‘A. Von Willebrand's disease B. Hemophilia A ©. Factor BC deficiency D. Acquired plateit defect B. Acquired vascular defect Answer: D~ the patient is most likely taking aspirin or other type of NSAID, The only Rx that could ‘correct this problem would bea platelet irnsfusion, which is certainly not indicated in this eae but ‘would be ina life-threatening bleed. a ‘Note: This material is copyrighted. All ights reserved. (Raward F. Goljan, M.D.) 2002, tems 3-4 ‘A 63-year-old man with urinary retention secondary to benign prostatic hyperplasia develops fever and chills shorly after insertion of en indwelling catheter. Fhysioal exam reveals warm skin and a Dounding pulse, Within 24 hours, he beains oozing blood ut of yenipanctre sites and from his ‘mucous membranes, Boshymoses appear over his trunk and extremities. Hig urine output desreases to <400 miLiéay. Laboratory studies reveal the following: Hgb 10 g/4L. (13.5-17.5 g/dL), WBC ‘count 2,000/mn! (4,500-11,000/mm), platelet count 140,000/zun (150,000-400,000/mnm'), PIT 42 sce Q6-d0 see), PT 18 see (1-15 sep), plasm fibrinogen 150 me/dL, (200-400 med) fibcin(ogen) degradation products >10 jg/mal (<10 pgfml.), D-dimers positive (negative), Blood gate ending sum BUN 30 mg (7-18 fl) sera seine 8 me. (06-12 e/a) "3. Which ofthe following apply o this eae? SELECT 4 A. Thwombotic thrombocytopenic purpura Disseminated intravascular coagulation ‘Consumption of voagulation factors Prerenal azatemia Secandacy fibrinolysis Decreased production of platelets Increased total peripheral resistance, Endotoxie shock “Answers: 1, CE, HE the paient has DIC snd iin renal failure (BUNVereatinine rato <13) 4° Which oft olowing isthe mos effentve extent for hipatn? Fresh frozen plasma Packed ed blood eels Psteict concentrates Lov dose heparin Antibiotics Homkyae neopep (Cayoprecipitate Answer; I: in DIC, the most effective Re is to Rx the cause of the DIC, in this ease, endotoxie shock secondary to Ecol. The blood components are als indicated, but essentially feed the fie and keep the patient alive until the underlying diseass is eradicated. 5. Which of the following laboratory test results are more often associated with classics! von Willebrand's disease rather than mild hemophilia A? SELECT 3 A. Normal prothrombin time B. Low factor Vill:coagulant activity C.Abnormal ristooetin coftctor assay 1D, Prolonged partial thromboplastin time E, Prolonged bleeding time Low VIM: amtigen G. Normal platelet count 1H, Response to desmopressin acetate Answers: CB, good comparison question Thema 6-12 Platelet count Bleeding time PTT er A. Decreased Prolonged Normal_-——Normal, B. Decreased Prolonged Prolonged Prolonged ©. Normal Prolonged Prolonged © Noxmal D. Normal Normal Prolonged Normal Normal Normal Prolonged Prolanged FE Nommal Normal Normal Prolonged G._ Normal Prolonged ‘Normal ‘Normal 6, A d-yrold child has eaten rat poison and is hemorrhaging 2 ‘Note: This material i copyrighted, All rights reserved, (Edward F. Goljan, M.D.) 2002 Answer: 12—rat poison is warfarin, which blocks all th vitamin K-dependent factors. Hoth PT and PIT ste prolonged 7. -A.$4-ycold man ison heparin to prevent deep venous thrombosis, Answer: E heparin enhances ATI, which neutalives most intrinsic factors and most of the final common pathway factors, hence both PT end PIT are prolonged 8, A 28-yrold man has been envenomated by a rattlesnake and is bleeding from all orifices and all needle stick sites Answer: B: the paticat has DIC and is consuming platelets, fibrinogen, prothrombin, V, VI 9. Achild has eaten row hamburgers and now has a bent anemia and rene fitare “Answer: As HUS due to 0157: 117 serotype of 2. coll Reniomber that ony platelets are consumed, not “coagulation factors 10. A man has a family history of a bleeding disorder which began with his mothers father Answer: D; hemophilia A, matoms! father transmits the disease f alt his aught’ (SXR trait) who ere -asymplomatie cates. The datightts transmit the gene to SOY oftheir sons. 11. ‘An afebrile &-year-olé boy presents with epistnis | week afler an upper repiatory infetion. Physical examination reveals scatters! peteohia and vechymoses over his trunk. There is no Iymphdcnopsthy or hepatosplenomegaly. A stool for occult blood is negative. His CBC exhibits & hemoglobin of 13 pA (12.0-15.0 g/.),loukooyte count of 8,50M/mm? (4,500-13,500/m) with ‘anormal differential count, an a platelet count of 10,000nn* (150,000—400, 000m). Answers A~ the patient has iiopatic thromboeytopenic purpura 12. A 38-year-old woman presents with fever, menial status alleations, snd epistaxis. Physical -exatination reveals retinal hemorrhages, widespread petechia and ecchymoses, anda postive stool ‘or ocent Blood. There is no lymphadenopathy or hepatosplenomegaly. The CBC report indicates the presetice of normal hemoglobin and leukocyte count, The eorectedzeticuloeyte count is 12%. ‘he peripheral smear exhibits numerous fragmented RECs, polychromasia, and s reduced number ‘ofplatalets. A bone mertow biopsy contains vascular channels containing platelet thrombi. There is 1 postive urine dipstick for blood and numerous RBCs are preset in the sediment. The serum ‘ood urea nitrogen is 40 mig. (18 mya.) and the seram ereatinine is 4 me/a. (0.6-1.2 smg/t). + Ac the patient has thrombotic thrembocytopenie purpura 13, An T&year-old smoking mle presents with deep venous thrombosis (DVT) involving the right ‘ower eal, He has a family history of recurrent DVIs and pulmonary crsboli in his mother and ‘maternal grandfather, His PTT and PT are notmal prior to reveiving a standard dose of heparin intravenously. The PTT remains normal 1 hour affer infusing heparin. The pathogenesis of his amos abnormal moat ely ela. adefieieney of anithromibin I 1 deficiency of a vitamin K-dependent factor antibodies directed against heparin Answer: C: the patient has hereditary ATU] deficiency. Heparin eanmot sncoagulate without ATIIL The ead i the lack ofinrease in the PTT with heparin. 14, The pathogenesis of hemorhagic skin necrosis associated with warfarin therapy is most closely essociated with. stbodicsdieoted against warfarin ‘drug hyperenstvty reaction ‘rmmane vasculitis secondary to warfarin protein C deficiency inthe patil ntidzombin I deficiency i the patient BEARD Pope B ‘Note: This material is copyrighted. All rights reserved. (Edward F. Goljan, M.D.) 2002 Answer: D: heterozygote carers for protein C become homozygote when given warfarin in ~6- hrs when the half-life of previously 7-earboxylated protin C disappears. Now the patient hhypercoagulable. ‘Blood Bank questions 1. A 65:year-old man in an intensive caro units recovering from surgery for a ruptured abocsinal ‘sorts meurysm. The patient required 12 units of packed red blood cells prioe to surgery in order to stabilize bis blood pressure. On the Sth postoperative day, he develops fever, scleral eters, and ‘ow back pain. Physical exam reveals seatered rhoneh throughout both lang fekts butno areas of ccnolidation. A rine sample taken from his indwelling urinary catheter exhibits pyuria. Cultures isolated coll. A CBC reveals a 3 g/ckop in his hemoglobin concentration when compared to his postopemtive levels, ditot Coombs testi positive. No hemoglobin is noted i-plasma. The “otal bilicubin is 4 mg/l. (0.1-L.0 me/AL) with w diet conjugated) ilirobin of 04 mal. (00-03 mg/L). ‘The serum alanine aminotransferase concentration is 20°U/L- (8-20 U/L). Which ‘ofthe follewing apply-to the patient's clinical condition? SELECT 3 ‘Febrile transfusion reaction Postransfusion hepatis Delayed hemolytic sransfusion reaction ‘Type IThypersensitivty reset Conjugated type of hyperbilirabinemia “Exvavasculan hemolysis ‘Answers: C, DF: the patent has had a delayed IVT due to an antibody ditested against an antigen on ‘the donor RBCs. 2, Ablood group G Rh negative worsan with negative antibody sereen delivers ablood group A, Rh positive taby. The baby is mildly anemic and develops an unconjugated hyperbiirabinemsia inthe First 24 hous. A ditect Coombs of cord blood returns weakly positive. Spheroeytes arenoted inthe peripheral blond smest. The baby's stool is grossly bloody. An Apt test ie performed on te stool ‘ample and reveals adit hemoglobin, The mother's antibody screcn is negative. Which of the following best describe the pathogenesis ofthe baby's anemia? SELECT 4 PRRARD “Type hypersenstviy reaction ‘Mother is nota candidate for Rs imme plobuin ‘ABO incompatibility protosts against Rh sensitization Positive direct Coombs is dus to anti-D coating babies RBCs [Newsora har a Gl bleed ‘Answers: A, B, C, B: ABO incompatibility does protect against Rh sensitization, bul se sil should receive Rh immune globulin, Mothers ani A,B IgG is coating the babies RBCs. The Apt text distinguishes fetal from al Hgh. The baby has swallowed mommies Blood and does not have a Gticed, 3. A.group 0, Rk negative S2-yearald woman develops fever of 108°F while receiving.a mit of O, Rh negative packed red blood cells aller a hysterectomy. A transfusion reaction workup on the ation revels the following: Sa PERED ‘Antibody screen negative Direct Coombs! negative Plasma normal color Urinalysis dipstick negative for blood retransfusion hemoglobin 7 g/l. (12.0-16.0 g/dL) Posttransfusion hemoglobin 7.5 gill. (12.0-1600 g/dL) Which of the following apply to this ease? SELECT 2 4 ‘Note: This material is copyrighted. All rights reserved. (Edward F. Goljen, M.D.) 2002 A. Donorblood contamination with Yersinia enterocolitica B. Type T hypersensitivity reaction C. Patient should-receive leakocyte poor blood for further transfusions D. Hemolytic transfusion reaction due to antibody destruction of donor RBCS B, _Pationsant-F1L.A antibodies are directed against donor leukocytes Answers: C, B: febrile reaction: patient must have been exposed fo blood products at some time in ber life for antibodies 1 develop (commen ia multiparous Women who commonly have fetomateal bleeds daring delivery) ‘An 82--0ld woman wih blood group A inadvertently receives blood group blood. He doesnot devsopahenoys wantion action, This is mos ily doe. ahaence of nohemaggtuttins with old age Saati car toy ©. Brutor’sagammaplobulinemi D. antithrombin I deficieney ‘E, _ahoont an-A JgM titer the donor unit “Answer: A: eldcly people lose thei sohemagelntinins, hence they may not develop hemolytic TRs even. 4 ‘with an ABO mismatch. ‘Which ofthe following is more often associated with Rh hemolytic disease of the newbom due to anticD antibodies rather than ABO incompatibility? SELECT 2 “A. No risk for hemolytic disease ofthe newborn during: fist pregnancy B. Positive direct Coombs’ on the babies cord RIC © Spherozytes in the newborn's peripheral blood D. Severe semis cequsing blood transfusion E. Unconjugated hyperblirubinemia ‘Type IT hypersensitivity reaction Answers: A, D; moter’ got sensitized ageinst Rh antigen in thir first Rh incompatible pregnancy, but 6 this does not affect that baby but future babies A blood group O, Rh negative 65 old-year-old man with known diverticulosis presents with 2 ‘massive lower GT bleed (hematochezia). Te has an estimated blood loss of greater thar 600 mL. cover the last few hous, An itravenous line with 0.9% normal saline-is in place, while blood is ‘being crossmatcied in the blood bank, His High is 6 g/L. Physical exam reveals cold, clammy skin, blood pressure of 70/40 mm Fig, and a weak pulse witha ate of 120 bpm. Owing to a shortage of , Rhinegative blood only 1 ofthe $ units of packed RBCs are O, RUinegative, while the remaining 4 units ate O, Rh positive. The patient has no history of «previous transfusion or exposure t blood. ‘products, Midway through the seeond unit of blood, which is the frst of the four mits of O, Rh positive blood, he develops wheezing, dyspnea, and swelling of his face. ‘The transfusion is ‘stopped, a subcutancous injection of aqueous epinephrine at a 1:1000 dilution ie given to the alien, and a transfusion reaction work-up is ordered, The transfusion reaction work-up on a post- ‘transfusion specimen of patient blood is as follows: Patient temperature: 10000 R Patient blood pressure: 60/40 mm Hg Patient pulse: 130 bpm, Patient plasma: ‘lear Patient antibody screen: negative Patient direct Coombs: negative Patient urine: negative diptie for blood ‘The pathogenesis ofthis patient's transfusion reaction is most closely related to... SELECT 2 ‘A. anti-D antibodies destroying donor D antigen postive RBCs B patient ant-HLA antibodies reacting against donor leukoeytes (Ca patent IgE-mediated reaction against a donor allergen D__atype L hypersensitivity reaction as ‘Note: This material is copyrighted. All rights reserved. (Edward F. Goljan, MD.) 2002 E_anintravascular hemolytic anemia Answers: C,D:allergie reaction 7. A blood group O, Rh negative woman with a negative antibody screen and no previous administration of Rh immune globulin during her prepnancy delivers a blood group B, Rh positive baby. The baby dovelops unconjugated kypebilrubinemia a few hours after bith, The pathogenesis of tho baby's jeundice is most closely related to. ‘maternal anti-D antibodies destroying the babies Rh postive RBCs intravascular hemolysis of fetal RBCS by anti-B IgM antibodies intravascular hemolysis of fetal RBCs by anti-A,B IgG antibodies extravascular hemolysis of fetal RBCs by ant 1g antibodies extravascular hemolysis of fctal RBCs by anti-A.B IgG antibodies Answer: E: the mother is ABO and Rh incompatible with her baby. She as not been previously sensitized to D antigen, soit cannot explain the babys fui, therefore, ABO hemolytic disease ‘of the ewbomn is the most likely eavse. 8 Whichofthe flowing characterize major srosmash? SELECTS ‘A. Prevents past-ransfsion hepatitia due to hepatitis © B. Does not guarantee survival of the infused donor RBCS €. Roles out the possibility ofa febrile transfusion reaction D B F. Pope Does not prevent patient antibodies developing agaist donor REC antigens Prevents antibodies inthe donor from destroying patient RACs Deteots the presence of patient antibodies against donor RAC antigens Answers: B, D, Fa major crossmatch involves mixing patient serum with donor RBCs to see if hey are © Blood group D.__Blood group O 9. A.AS.jearcld man with exertional éysmea and extome fatigue and along history of duodenal ter disease is noted to have dark black, try stools. Physical exam revels pale conjunctiva and» wide pulse presse. Laboratory stafies reves! 26 hemoglobin and «low MCY. Donor blood {or icansfasion ofthis patent based on te pe of peptic uler disease he has would most key ave hic of the above forward and back ype els nthe Blood balk? Answer: blood group © people can ony recive Hood group. Remember thatthe forward type with ‘satis identities the blood group, while the back type using text RBCs, identifies the antibody ‘tvociated with he blood grow. 10, A32yeurold woman and her husband have 10 eidren. Her husbands Mood group is A. Al of thee dhiren sre ether A, B, or AB. In onder to explain the blood groupe inher hire, yO otidengzl the wou 6 ave wich fi above ara ak ope es om se oF ‘Answer: B: she must be AB to have A,B, er AB childcen, AB peopl are univers recipients since they The antibodies. LL, Ad?apeanaold ar a gastce adenocarcinoma. Based on his diagnosis, which ofthe above blood troup tpesis bert for tastsing the pater? ‘answer: Ar gat adenocarcinoma ae ously een ingroup A patents tis abways best to give the tient the same ABO group rae tan Orpacked RECs 12, ‘A patient wit this bleed group could develop a bemolyictransfision seastion after receiving ABS tod or A bod Answer: the patient mast be blood group B (Note: This material is copyrighted. AI rights reserved. (2dward F. Goljan, M.D.) 2002 13. A phlcbotomist inadvertently sticks himsolf with « needle after drawing blood ftom 2 patient with ‘AIDS. Which ofthe following infections isthe phlebotomist at mos ick for contacting? A HV B Hepatitis A C, Hepatitis B ‘D. Hepatitis ¢ E, Syphilis Answer: A: 1:300 chance of becoming HIV positive 14, A phlebotomist inadvertently sticks himself with a needle after drawing blood fom a patient. ‘Which ofthe following infections is the phlebotomistat most risk for contracting? A HV B HepatitisA Hepatitis 8 D. Hepatitis © Syphilis Answer: C: HBY has the greatest viral load in blood of all vises. You cannot: get syphilis from. transfused blood. 15. Which of the following is the most common antibody encountered in clinical practice? A, AntiHAVIg@ B Anti HBs C AntHHCV-Ig D. Ant-CMv E. Htecophile antibodies Answer: D: it ia almost impossible to find blood that is negative for antibodies against CMV. "This was ‘werified withthe National Red Cross 16. A patient who has chronic hepatitis and hus heen transfused in the pust most likely hus eatibodies iirected aginst. A, hepatitis A B. hepatitis B © hepatitis € D. HIV B CMY Answer: C: HCV is the MCC of iransfusion hepatitis ‘Cardiovascular questions 1. A 72-year-old man presents with a sudden onset of Jot flank pain, Jn the emergency root, the patient is hypotensive. A pulsatile mass is palpated in the abdomen. Which of the following is ‘MOST reeponsble forthe pathogenesis ofthis patent's eandition? AL Atherosclerosis B.A defect in bitin CA defect in collagen D. _Long-standing hypertension E. Immune desteuction of elastic tissue Answer: A~ ruptured abdominal aortic aneurysm. Note the rupture triad of left flank pain, hypotension, ‘and pulsatile mass "Note: This material is copyrighted. All rights reserved. (Edward F. Goljan, MD.) 2002 2. A42-yearold man with a history of curdise death in his family presents with a audden onset of ‘severe, rerostemal chest pain with redition of the pain into his back, His left pulse is absent. A. high pitched diastolic blowing marmur that increases with expiration is heard immediately after Ss. “There is widening of the artic knob on a chest xoray, The mechauisin for this patient's condition is MOST CLOSELY related to... A. atherosclerosis Ba defectin fibrin © adefectin collagen 1D, aminfeotious process E. — anacute myocardial infarction Answer: B- dissecting aortic aneurysm. Note the type of pain radiation into the back, absont plo, ‘murmur of aortic regurgitation and widening of the aortic knob, 3. A 26-year-old woman presents with a history of chest palpitations particularly whea. anxious. Physical exam reveals « mid-systolic ejection click followed by a murmur. The elie nd murmur move eloser to $; when the patient is standing and closer to $2 when the Tying dows. The ‘mechanism for this patient's valwlar disorder is MOST CLOSELY related to. a defeot in fbeillin PEORP {defect in collagen sytsis Answer: D- mal yalvo prolapse. When the LVEDV is desreased (standing, amet) the cick and ‘murmur come closer to ST. When the LVEDV is increased (lying dows, clenching fst), the click ‘and murmar come claser to $2 4. 428:yearold patient has «family history of sulden cardiac dea ta young ag‘The patient has 2 stl jcetion murmur that decreases in intensity when the patente ying down and increases in intensity when standing up. An échoeatdiogram reveals ebnocmal roovetcat ofthe antetior mil valve leaflet against an acyzametrically thickened interventricular septum. The patient MOST LIKELY has. ‘A. mitral valve prolapse 1A hypertophic cariomyopathy ©. congenital bicuspid aocic valve D. cardiac myxoma of the Tf atrium B. _infetive endocarditis voving the mitral valve Answer: B= kyperzophic eariomyopaihy MCC of sudden death in young people. The arymmetic ‘septum has abnormal conduction bunds, 5. A 65.jearold man on the Sth day of hospitalization for an ace siterior myocardial infarction has secusrence of chest pain and un increase in both CK-MB and iroponind and T. The patient MOST LIKELY has... ‘A. papillary muscle dysfunction BL aright ventricular infarct © aventricular aneurysm, D. amyocardial rupture Se 6 Which ofthe following is present in BOTH left and right-sided heart failure? A. Spheart sound B. Pillow orthopnea Pulmonary edema D. Neck vein distention E, Dependent pitting edema 28 ‘Note: This material is copyrighted. All rights reserved. (Paward P, Golan, MD.) 2002 Answer: A~ choice B in LAR, choioe Cin LHF, choice D in RHF, choice Ein RHF) 7. You would expect a patient with an atrial septal defeot 10 have which of the following oxygen saturation (Sa0,) findings obtained by cardiac eatelerization? Norinal Patient Patient Pationt Patient Patient S20,% A eo anen a Rightatrium = 75 B : a | Rightyentricle 75, 0 o 3 7% 80 Pulmonary at 75 80 5 © 4 Pulmonary vein 95 95 9 95 SS Teftventrile 93 95 9 «9580 Aorta 95 98 9 95 8D S10; oxygen satorafion ; 4.” Answer: B~ isan ASD; A= VSD, B= ASD, C=?DA, D= Tetralogy, B = transposition. Note that fa step up im oxygen saturation in a chamber or vessel usually means a lef to right shunt, while a step dovin in. oxygen saturation indicates aright to let shunt (eyanotic CHD) 8. A 20-year-old man with a viral myosarditis who develops iiypatension, netic vein distention, a dep Seto ecm hte od med bare A 1B. constrictive pericarditis © typovalemic hock D. 2 porcanda effsion B, a distesting sonic aneurysm ‘Answer: D~ Beck’ triad of pericardial effasion. First step i to do-an echocardiogram, If postive, then 2 pericurdioventess must be done. ‘tems 9-11 ‘Aortic stenosis ‘Aortic regurgitation ‘Mitral wenosis ‘Mitral regurgitation ‘Tricuspid reguritation 9. A 70-year-old man wit diminished pulses anda history of angina and syncope with exerise has an ‘ejceton type murmur radieting into the carotid arteries ‘Answer: A~ aotic stenosis isthe MC valvular lesion asociated with angina end syncope with exercise, Its ao the MCC of microangiopathio hemolytic anemia, 10, A 58-year-old man with left-sided hear filore has an $3 and $8 heart sound dod « pansytoic ‘muri located the apex that increases with expiction Ansofer: D~mltral regurgitation in heat fun dv to iretching of the mitral valvering de to volume ‘verload in the left ventricle 11, 29-year-old intraveious drug abuser has fever, giant c-v wave, ands pansytolie marmur along the left stemal border that increases with inopiration Auster: E~ tricuspid regurgitation isthe MC valvular lesion TVDA. Is eto 8 aureus. Remember that, sTivight sided murmurs end abnormal heart sounds ineease oa ispiration, while the revere rue or let sided marmurs and abnormal heart sounds, 12, Which of the following types of hepatitis is associated with vasculitis due to classical polyarterts nodosa? Hepatitis A Hepatitis B Hepatitis Hepatitis D Hepatitis B PpOR> Poppe 2 ‘Note: This material is copyrighted, All rights reserved, (Edward F. Goljan, M.D.) 2002 Answer; B- polyarteriis nodosa. High association with HBsAg. It is not associated with p-ANCA (microscopic polyangits is). c-ANCA is seen in Wegener's granulomatosis. 13, A Tyearold bay presents with a low-grade fever, mralzia, colicky abdominal pain, and a palpable purpric rash fiited to the lower extremities. Laboratory studies reveal a gusiac-positive ‘Stool, «urinalysis wth red blood cell (RBC) eats, hematuria, and-ild proteinuri, and a CBC with ‘s normal I, Ht, and platelet count. Which ofthe following isthe most likely diagnosis? 1 ienipeeyomaens Guy B. Systemic ©. Posttreptosoceal glomeruloneplatis D, Rocky Mountain spotted fever . Henoch:-Schénlein vasculitis ‘Answer: T= it is a type Ill inimunocomplex disease. Most clinicians think that HSP and IgA ‘lomensloephiitis ae te-same, Respiratory questions 1. In which of te following clinical scenarios involving pation with ang disease would you expest ‘pulmonssy fonction studies to exhibit decreased compliance, increased elasticity, and am increased FEV ad PVC rt? ‘A. Geyear-old child with reowrrent respiratory infections and stectorthes B. 28-year-old non-smoking male wit bilateral lower lobe emphysema ©. S6-sear-old smoker with productive cous, dyspnea, and cyanosis BD, 10-year-old gil with bronchial asthe requiring system steroids 1. _30;jeanald dyspnoie Afioan American with iletera hilar nodes Answer: B- the PFTs describe a restrictive ung disorder, in his ease, sireoidoss. Choice A destibes ‘etic fibrosis, B describes el-antitrypsin deficiency, C desribes COPD, and D describes ‘bronchial asta 2. _A.45-year old worman 24 hours post cholecystectomy develops fever and dyspnea. Physical exan ‘eveais decreased percussion, Snsreased tactile fremitus, and desreased breath sounds in the rl lower lobe. ‘The diaphragm is clevated and there is inspiratory lag on the right sid. The patient ‘MOST LIKELY has... ‘A. atelectasis B.adungabscess C.bronchepncumonia D. apulmonary inficetion E. _aspontmeous pneumothorax Answer: A~ these ae the classic physical sindings of atelectasis. They are similar to consolidations noted, {in bronchopneumonts and lobar pneumonia. Remember that fever in the first 24-48 his after ‘surgery is atelectasis. ‘én afebrile 2%-yearold man develops a sudden onset of left-sided, stabbing. chest pain with dyspnea. Physical exam of the lok chest reveals hyperresonanee to percussion, deviation of the ‘tachea (o the let, elevation of the diaphraum, devreased fuctile fremitus, and decreased breath sounds, The MOST LIKELY diagnosis. ‘A. pleural effsion B. bronchopneumonia C. ‘tension pneumothorax D.apulmonary infarction E, spontaneous pneumothorax Answer: Ein spontaneous pneumathorax, the lng collapses esusing the pressure in the pleural cavity to ‘be the same as that inthe atmosphere. A tension pneumothorax is # tear in the pleura, which opens and admits sirinto the pleural eavty dusing inspiration and closes during expiration. This increases 30 "Note: This material is copyrighted. All rights reserved. (Edward F. Goljun, M.D.) 2002 ‘the positive pressure in the pleural cavity and shifts the mediastinum and trachea tothe contralateral sie. Also occurs when ascending fo the surface in seuba diving ‘A.newbom child develops dyspnea, tachypnea, intercostal muscle retraction, and cyanosis 4 hours afer bith. The mother developed gestational diabetes mellitus end was in poor glycemic control ‘hroughout the pregnancy. A chest x-ray reveals a “ground glass” appearance in both lnngs. The ‘primary mechanism for this patient's respiratory problem is A. spiration of anmfotic uid 1B. group streptococcus pneumonia. C. —decreased production of surfactant D. Chlamydia trachomatis pneumonia. E, _ hear failure from congenital beart disease ‘Answer: C- fis is lassie RDS. Poor glycemic control inthe mother leas to hyperglycemia inthe fetus 5. and release of insulin, which inhibits surfactant synihesis by type Il pneurnoeytes. Which of the following deseribes a poeumonia due to Mycoplasma pneumoniae rather than ‘Streptococcus pnesanoniae? A. Highiever B. Insidious onset. ©. Productive cough D. Increased tactile fiemitus E. _ Neutrophilic leukocytosis ‘Auswer: B- this question compares atypical with typical pneumonia, the former most often due fo Mi ‘pneumoniae and the later S. pneumoniae, The former isan intetstitfal pmeumonia and the latter a ‘consolidation type of pneumonia, All other choices are these of typical pneumonia andl would not be expected in.an atypical pneumonia 6. A.S8-yenold smoker presents with weight lors and cough, Physical exam reveals a mild id lag on the left and s-pinpoint pupil, scatered sibilant rhonehi troughont all ung fields that clear with ‘coughing, and an increased anteroposterior dsmeter. Based on these findings, you suspect the patient has. A. aPancosst tumor 1B. a thornco outlet syndrome © the superior vena caval syndrome D, obstructive lung disease without primary cancer 1B, _ obstructive lung disease with metastatic cancer from another primary ste “Answer: A~ a superior sulcus tumor, usally squamous cancer, invades the lower part of the brachial 7 plexus std destroys the superior cervical ganglion leading toa classic Horner's syndrome. ‘A 65-year-old man with urinary retention sccondary to prostatic hyperplasia, develops spiking ‘ever, and tachypnea, Physical exam reveals intercostal muscle retrctions ahd bilateral inspiratory crackles. A chest xray exhibits bilateral interstitial and alveolar infrates. ABGs demonstrate severe hypoxemia. You expect the blood culture reveals. A. gram positive diplococei 1B. gramnegative diplococei C. gram positive cocci D. _gramnegative rods E,__ram positive rods ‘Answer: D~ the patient has gram negative sepsis due to Ecol gram negative rod) which progressed into ARDS. Indwelling urinary catheters are the MCC of this type of sepsis. ARDS and DIC aro ‘complications of gram negative sepsis. Inspiratory stridor is commonly associated with ‘A. arespirstory syncytial virus infestion 1B. a parainfluenea virus infection C. —aspirin-induced asthma 3 ‘Note: This material is copyrighted. Al rights reserved. (Edward F. Goljen, MD.) 2002 D. — shinovinas infeotions E. _ choanal atresia Answer: croup oF laryngotacheobronchitis is due to paranfiuenza vies, It eads to obstruction inthe teachea, which on x-ay appear like a "steeple of church, Acute epiglttis due fo H.iluenzae aso produces inspiratory atidor, 9. GHlanyeia trachomatis and the respiratory syneyil virus are BOTH commonly associated with A an interstial ype of pneumonia 1B. _lagngotracheobroncits Croup) C.therespirenry diares gnome D. typical community-acquired pneumonia . _hospital-acqited(aosocomial) pneumonia ‘Answer: A RSV is the MCC of pneumonia snd bronehioiis in children. CL wachomats produces & Staccato cough, wheezing with trapping of at, td eosinophilia. These is no Tever and bilateral conjunctivitis is usually presents well 10, Which of the following is more often associated with Kichsttta pneumoniae than Pseudomonas ‘aeruginosa’? A. Upper lobe cavitation B. Green-colored sputum C. Association with eystic fibrosis D. Association with respirators B. Productive cough Answer: A~ choices B, C, D are features of P. aeruginosa. Both have productive cough (choice E) 11. Ine 30-year-old msn who lives in Tennessee, you would expect a calified solitary coin lesion in ‘the lang to epresent... A. a foreign body B. snold gramloma metastatic cancer D, primary lung cancer B _abronehial hamarioma Answer: B= most solitary nodules in the lungs are granulomas ad in the Ohio-Teanessee valley, ‘histoplasmosis is the most likely cause. In the Southwest it would be eoccidioidemyeosis, in the Southeast, blastomycosis, and in the Northeast, eryptococcosis, 12, A'55-year-old non-smoking coal worker bas arthritis and nodular lesions i se lungs. His'PPD akin ‘test is negative. You suspect the patient has... primary Tung cancer E. miliary tuberculosis ‘Answer: B this underscores the association of sheumatoid nodules in the lungs with a pneumoconiasis, {inthis case coal worker's pneumoconiosis. 13. Ina G2-year-old man who has been a roofer for 25 years and a smoker for 10 years which of the following cancers would he be most likely prone to developing? A. Pleural mesothelioma 1B, Primary lng cancer ©. Laryngeal careinoma D. Oral eancer E, Pancreatic cancer ‘Answer: B- the patient has had asbestos exposure, since roofing material at that time had asbestos in its composition. Whether the patient was a sipoker or not, a primary lung eancer would be moce common than a mesothelioma. Mecotheliemas take much longer to develop than hing cancer and 2 ‘Note: This material is copyrighted. All ights reserved. (Edward F. Goljan, M.D.) 2002 hhave no smoking relationship. The MC lung problem associated with asbestosis a henign pleural plague. Ms pn nti, Ney Yok er onary te, Wh of he owing ‘soups of pathogens are on Your differential st? ‘A. Histoplasma capsulatum and Cryplocoocus neofoamans 1B, Ceyptocoeens neoformans and Cocediidesimmitis CC. Blastomyecs dermatitis and Histoplasma capsulatum D. —Aapergilus fumigatus and Coceidiides ims E _Pacunooysis cavni and Aspergillus fumigatus aor A, elms iin Copia revs, Sung ary Horas carry Cryplococcus and bot zoos on bridges. 1, Which of th following ea bypectenstvypocumonte tha primarily ours is texte works? Ap Silofilers disease B. Bagassosis © Farmers ung D. Byssinosis B, Sarcoidosis Answer: D- exposure to linen, hem, or eoton products may cause 2 hypersensitivity eaetion i the lung ‘with dyspnea and infiltrates. Patents. ireprove over the wecktad and get depressed on, Monday “inowitg that hey wil be sick, ene the ferm “Monday morning blues’. 16, Which of the following is « hypersensitivity pneumonitis that commonly oceurs in farmers who ‘enter a closed roam with fermenting carn? A. Silo fillers disease B Bageesosis Farmer’ lung D. _ Byssinosis B, Sarcoidosis “Answer: A. Silo fila’ disease is rlated to nitrogen dioxide is given off bythe fermenting eo. Tcauses ‘a itmediate reaction in the hangs. Farmer's ng is dbe to a hypersensitivity reaction against ‘thermophilic actinomycetes out in the fcids. Bagassoss isa reaction against sugar eane. 17, IF x ptient is breathing room sir (21% oxygen), the erterial PCO, is $0 mm Hg, and the arterial ‘PaO is 40 mm Hg, what is the Alveolr-artrial (A-a) gradient of the patient? A 10 BS c a Db. 2s B30 ‘Answer A-~ the formula to caloulate the PAO; is PAO, = % O, (713) - PaCOO.8. In this problem the PAO, = 021 (713) - 800.8 = 50, therefore, A-a = 50 - 40 = 10, A medically significant A-a gradient is > 30 mm Hg. Any lung disease would be expected to increase tho gradient~ o., ‘pocumonia, COPD, sarcoidosis, ARDS. Hypoxemia due to conditions not direetiy involving the Tangs have a normal gradient. For example, a ‘Note: This material is copyrighted. All rights reserved. (Bawaed F, Goljan, MCD.) 2002 ‘Gastrointestinal questions Items 1-6 A. “Vibrio cholera BL Bacillus cereus © Shigella sonnet D. Salmonella typht Campylobacter jejuni E, Yersinia enteroealiies G. Staphylococeus aureus HL. Enterotoxigenie colt 1, An afebrile 22-yearcld man and several ther members of his family developed severe vomiting ‘without diarrhea ~1-§ hours aflerenting potato salad ata picnio. They all recovered uneverflly 12-24 hous ater. Answer: G- this i doe to S aureus producing toxin which is ingested in the food, Caltre the Food, sot the stool 2. A 23-year-old man developed explosive, watery diarrhea with blood, leukocytes, and meus ~3 ays after eating chicken that was improperly cooked. Comma to S-shaped organisms are noted in. the fecal sm of stool along with RBCs nid lekooytes. Answer: E Campylobacter is the MCC: of fevasve gastroenteritis inthe U.S. It oan produce: crypt shscesses and simulate UC. 3. A febrile 10-year-old child presents with sever right Tower quadtant pain thats interpreted by the attending physician as acute appendicitis. At laparotomy, the surgeon notes thatthe appendix is octal. However, the mesenteric Iymph modes are markedly enlarged and have. focal areas of microabseess formation on cut section. Answer: F= Yersinia is cause of mesenteric adenitis, a granulomatous disorder; tat ean be confused with acute appendicitis. 4, -A.29-year-old man develops watery diarthea and volume depletion shorly afer visiting the Gof Coast states “Answer: A~ Fibro cholerae produces a secretory diarrhea duoto toxin stimulation of ¢-AMP. 5. Ona tip outside of the country, aman develops a high fever associated -with bradycardia, absolute nentropenia, and splenomegaly. A blood cultre is positive for « graze negative rganiem. Answer: D- Salmonella ypRt produces this classic triad during the cond week of infection, Auman reservoirs invalved. 6 A23-yr-old man develops vomiting and diasea after eating refried vice and tacos, Gram positive rods are noted in the stool. “Answer: BB. cereus produces a preformed toxin. Salmonella enteritidis is the MCC of food poisoning Tams 7-8 A. Yersinia enterocolitica B. Staphylococcus aureus © Eaterotoxigenic F col? D. Cryptosporidium parvum E, Mycobacterium avlum-rtraceltulare 7. A 2Byearold man with ATDS presents with chronic, reemcent, profuse, nonbloody, watery diarrhea. An Entero-Test (sing test reveals oocyst that are partially acid-fast positive, Answer: D- Giypiospardium, a spororoan, i the MC pathogen in AIDS diathea 8 A 25-year-old medical student daring Spring break outside the country develops fever, vomiting, atxdominaleramps, and watery ama ~14 hours after eating a few tacos purchased from a srt vendor. He recovers uneveatflly in 48-72 hours. Answer: C- itis sceretory dares duc to heat table toxin that stimulates guanylate excise M4 ‘Note: This material is copyrighted All rights reserved. (Baward F. Goljan, MiD)) 2002 9. “Answer: B- Candie isthe over MC fimgus infection in AIDS. Fsophagiis s ATDS-efining but not thrush 10, An afrile 25.ear-okd medical student presents with iatemitent complaints of left and right ower quadrant abdominal pain and distention associated with alternating periods of mucoid icra and constipation, He sates tat slooking rlicwes the pain, flexible sigmoidoscopy aed ‘ool music exam are boi normal. The patent MOST LIKELY has, ‘A. aniatrinsc bowel moty disorder 2 inflamtcntory bowel disease chronic appends D. — mslanosis coli E. celiac disease Answer: A initable bawel syndrome 11, Which ofthe following correctly describes a gastric rather than a duodenal uloer? ‘Association with Helicobacter pylari ‘Highest incidence of pecferalion ‘Small risk for adenocareinoma. Pain awakens the patient at night Association with Zollinger-Elison syndrome Answer: C- both are associated with H. pylort. Duodenal ulcers have the greatest asoefation with I ‘pylori and also have the highest incidence of perforation and an association with ZE in MENT 12, The MOST COMMON loeton for vet, ply, ad cance nthe gatos! acti FUpE> A ascending esto B. sigmoid colon esophagus D. stomach E.restum Answer: B-selfexplanatory 13. Which of the following characterizes ulcerative colitis rather than Crokin’s disease? A. Diseontinuous spread B. Toxic megacolon ©. Fistula formation 1D. Perianal disease B, Obstruction Answer: B~all the other choices characterize CD 14, A 38-year-old Asian woman has along history of explosive diartiea und abdominal distention aftr ‘eating dairy products, The pathogenesis ofthis patient's diarrhea is MOST CLOSELY related to. A. antigiadin antibodies B. activation of eyelic AMP. CC. infrautninal esmotically active solutes 1D, nmooval injury with increased permeability B. _ loss ofthe absorptive surface af the emall bowel 7: C- lactase deficiency. Lactase is a disaccharidase and a brush border enzysne, It produces an ‘osmotic type of diarthea, Answé 35 ‘Note: This material is eopyrigited. All rights reserved. (Edward F. Goljan, M.D.) 2002 15, A Goyer old man smoker presets with weight los, « dragying sensation in his right upper ‘quadrant, nd crampy left ower quadrant ebdominal pan. He bas alicmating bouts of constipation and diathea. In adstion, he states that blood coats and is mixed in with his stools. He sometimes hn pain with defecation. There is mild hepatomegaly, The rectal exam reveals non-thrombosod extemal hemorthoid. The sion i gulac positive. A complete blood cell count (CBC) reveals « ‘il microti anemia. The MOST LIKELY ease for this patient's condition is... A. angiodysplasia Banal fissure C._diverticolitis D. hemorhoids E, colon cancer Answer; B~there is also evidence of liver metastasis 16. rol man preseais with Mushing of the face, watery diaea, weight loss, and multiple fn his liver. The primary site forthe canoer that led fo the above symptom complex is tong fiver stomach appendix terminal ileum Answer: - the patient has the eareinoid syndrome, Appendiceal carcinoids do not metastasize to the liver, bo it would not produce the syndrome, 17, Hlematochezia in elderly patients that eznnot be identified with a barium enema study is most “commonly due to disstdetlocsted i the. POOH PRES A rectum B. stomach ©. sigmoid cofon D. proximal duodemusa E. cecum and aseanding colon ‘Answer: E the patent has angiodysplasia, There is ¢ relationship with aortic stenosis and von Willebrand’ disease, Diverticulosis is the MCC of hematochezi, however, the barium enema is orm 18, A febrile 58-year-old man with along history of chronic constipation complains of «steady pain in the left lower abdomen, He has had frequent atacs of pain in these area fr several montis and ane episode of bloody stools that spontaneously resolved. Physical exam reveals rebound tendemess and a palpable mess inthe let lower quadran. A stool quai is negative. A CBC exhibits absolne neutrophilic eukncyosis and Tet sil A flat plate is reported o show no at under the diaphragm, The most kely disgnosis is. A. vobmlua B.coloncancer ©. ischemic colts D. acute diverticulitis BE. _ small bowel infetion Answer: D- acute ivertculiis presents as a "ef-sided appendicitis” 19. A S8.yr-old woman complains of epigastric pain, weight loss, vomiting of coffee ground-ike ‘atrial, and der black sticky stools. Physical exam revels epigastric pain to deep palpation and non-eader, hard left supraclavicular lymph nodes. Both ovarie ae enlarged and fam on basal pelvic exam. Which of the following seenarios best explains the signs and symptoms én this patent? ‘A. Stomach cancer with metastasis to the ovaries B Primary ovarian cancer with metastasis to the stomach 36 "Notes This material is copyrighted. All rights resorved. (ward F. Goljan, M:D.) 2002 © Primary pancreatic cencer with metastasis tothe ovaries ‘D. Malignant lymphomna ofthe stomach with metastasis to the avaties E. _ Primary ovarian cancer with metastasis tothe supraclavicular lymph nodes Answer: A~ ole Virchows node and the history of weight loss. Remember that « postinenopausal 20, ‘woman should not have palpable ovaries. It could by primary ovarian eancer, or ae in this case, ‘metastasis to the ovaries (Krukeobrg tumor) from stomach cancer, A 65-year-old man with a chronic acthythnia with an jregularly iregular beat presents with a ‘sudden onset of severe abdominal pain associated with vomiting, abdominal distention, and bloody iarthea. Physical exam reveals hypotension, absent bowel sounds, uo rebound tenderness, and a guaine positive, liquid stool. A eatnplete blood cell count exhibits a white blood cell eatmt of 35,000 cellsfil. with an absolute neutrophilic leukocytosis and left eh. The serum amylase is ‘elevated. The most likely diagnosis in this patient is. ‘A. ischemic ealis B. hemorrhagic penercatitis ©. acute appendicitis with perforation. D. acute dveticulis with perforation 1B. _hemorrhagio infiretion of tho small bowel ‘Answer: E- ail Srilation js the MC acthythmia associated with embolization, in this ease, to the 2, superior mesenteric artery. Amylase is present in the small bowel, so do not be fooled into calling, {his hemouriapie pancreatitis, ‘A €2-year-old woman complains of increasing pain and difficulty swallowing liquids and satis ‘over the past several months. She has lost 15 pounds in the past three months and has occasionally ‘experienced acute substemal pain and regurgitation of food into her mouth wien lying down at night. Esophageal manometry reveals aperistlss ofthe ower esophagus and increased pressure in the lower esophageal sphineter (LES). Which ofthe following apply to this ease? SELECT 2 A. Elevation in ant-centromere antibodies A. _Involves stated muscle in upper esophagus © Absence of ganglion cells in the LES myenteric plexus D. Bird's beak appearance on barium study. B. Adenocarcinoma ofthe distal esophagus ‘Answers: (C, D- the patient has achalasia Hepatobiliary/pamcreas questions a A febrile 12-year-old boy with viral infection lapses into coma, Physical exam reveals pepilledema and hepatomegaly. The serum ammonia and transaminases are elevated and ihe PT is ‘prolonged. The patent MOST LIKELY has.. A. viral hepatitis B. Reye's syndrome C. salicylate intoxication D. acetaminophen toxicity E. _ am antitrypsin deficiency Answer: Bvariella or influenza infections + salicylates may lead to Reye's syndrome 2. ‘Which of te following serologic data best represents a patient who has recovered from hepatitis B? HBsAg HleAg Anti-IIBe-lgM Ant-HBelgG AntlITBs ‘A. negative negative negative negative B. positive positive negative negative negative negative positive positive D. negative negative negative positive a7 Note: This material is copyrighted All rights reserved. (Edward F. Goljan, M.D,) 2002 Answer: C- A= serologic gap: not infective since HBeAg and HBV DNA are absent, B = acute or chronic HBV, D = vaccinated: note the difference from someone who has recovered there is no ‘loody diate ad ight upper quadrant pai, The patient MOST LIKELY bas. A anebiasin echinococcosis GC. peutecoleayeite D. —escending cholangitis E. _tyetatate colon cancer ‘Answer: A- Rr is metronidazole 3h 48-year-old aloholio with cross and chron pancreatitis as sleatorthca and prolonged Pt. ‘The FT docs ot cotet 9 nonal afer giving an fntwusculaeijeation of wien K. You once thatthe patient MOST LIKELY hs A wtamin K deficiency Bs eicaatngsaicoegulant {C.__amisoated factor deficiency D. _fnidesuate synthesis of conglaton factors TE _ viwmn K deficiency eccondary to malatorption “Answer: D- if the PT ad coneced, thn the ptint hs vitamin K deficieney, whic, inthis patient, could be dve to malabeorton of K fom chronic panereatis or blest deficiency related to his cihoss 5. A 45-year-old man presents with increased skin pigmentation, steatorrhea, and diabetes mellitus. “The nechaniam MOST LIKELY responsible Tor tis constellation of findings. A. amyloidosis B. aleoholic cithosis © adefectin iron metabolism D. alphayanttrypsin deficiency E. _adefect in coppermetabolism Answer: C- the patient has hemochromatosis, an AR disease with inreased iron absorption from the ‘stall bowel, The tem "bronze diabetes” deserbes the skin pigmentation and DM tron in ase generates hydroxyl FRs which damage the tiswue- cirhosis, chronic pancreatitis, restrictive Cardiomyopathy. Serum fern is the best screening test. Hepatooclolar carcinoma isthe MC cause of death, © Hh nie of he fooving ett wad you expect «conus 780% of the ttt A. Gilberts smacome DB. Chronic viral hepatitis ©. GrigheNaijar syndrome 1D. Stone inthe common bile duct ._ Extravasoular hemolytic anemia Answer: D- CB »50% always indicates obsructve jaundice. Choice A (Gilberts) would have a CB 20% (decreased uptake and conjugation), choi B would have a CB 20-50% (problem with ‘plakelconjugation aad necrosis of bile dicts causing release of CB), choice C (Crisle-Naijer) ‘would have a CB <20% (deficiency of conjugation enzyme), choice B (EA) would have a CB 20%, due to macrophage destrition of RHCs and UCB asthe end product of Hig degradation. 7. An afebrile 62-year-old man wilt a history of alcoholism, smoking, end ehronic pancreatitis resens with weight loss, a slow onset of painless jamdiee, and @ normoeytic anemia. Phys {xamreveas a palpable gallbladder and elight-oloed stool. The patiemt MOST LIKELY bas. ‘A. hepatocellular eaeinoma 1B. carcinoma ofthe gallbladier 38 ‘Note: This material is copyrighted. AIl rights reserved (Edward F. Goljan, M.D.) 2002 ©. primary sclerosing cholangitis D. A stone in the eommon bile duct FE. _ carcinoma of the head of pancreas ‘Answer: E~ note the obstructive signs of light eolored stool and palpable GB (Courvoisir's sign) ‘A 38-year-old woman presents wit chron liver disease, greenish-hrown deposits in the limbus of the eye, and chorenathetotie movements. Expected findings inthis patient include. SRLRCT 2 A. high total copper levels BL increased percent iron saturation C.antisICV IgG antibodies D. low ceruloplasmin levels ‘E, _ degenerative disease in the lentioutar nuclei ‘Answers: the patient has Wilson's disease, Not that total copperis low because ceruloplasmin, the 9. ‘binding protein for copper is low. The Kayser Fleischer ring is desaribed ‘A SS-year-old woman presents with generalized pruritus, non-ender hepatomegaly, and yellow papular lesions scatiered over her skin, She:has marked elevation of serum alkaline phosphatase 4nd ygtamyranaiese, anormal ttl bili, slg elevated serum taunt, and ¢ severe hyperlipidemia, An endoscopic retrograde cholaugiopanereatography (ERE) study ofthe ‘cominon bile dict is negative for stones: Which of the following are expected findings in this patient? SELECTS Hypectriglyeeridensia Biliary citrosis Antimitochondrial antibodies Inereased serum IgG levels Granalomsious inflammation. REO Bp Answers: B,C; I~ the patient has primary biliary eishosis, where there is yranulomatous destruction of 10. bile ducts in the portal triads. IgM levels are inereased 4s well as cholesterol due to abstraction to ‘ile, ‘Which of the following complications isn direct by-product of portal hypertension sesoudary to alcoholic cinhosis? SELECT 3 AD Ascites B. Gynecomestia C._Beophagee! varices 1D. Hepatic encepialopathy E. Peviutbilical caput medusae Answers: A, C, E~ gynecomastia is due to estrogen excess and cacephalopathy due to an increase in ammonia and fase neurotransmaitters ‘A.25year-old wouan develops jaundice. The conjugated bilirubin faction is > 50%6 of the total Diliman and the serum alkaline phosphatase is markedly elevated. You suspeet the patients taking whith of the following medications? SELECT 1 A. Isoniazid B. Allopurinol C. Tetracyetine 1D. Acotuminophen 1K, Oral contraceptives ‘Amswer: E— birth control pills produce intrahepatic cholestaais 2. ‘An afebrile high schoo! wrestler develops jaundice, light colored stool, dark urine, and mild non- ‘tender hepatomegaly, ‘The urine is negative for urobilinogen and positive for bilirubin. The serum alkaline phosphatase is markedly clevated, His hepatitis serology test and heterophile antibody test are both negative. You suspect the patient has. SELECT. 1 ‘A. astone in the common bile duct Been taking anabolic steroids 9 ‘Nofet This material is copyrighted. All rights reserved. (Edward F. Goljan, M.D.) 2002 C. _infeotious mononucleosis D, non A, non-B hepatitis E._ ehwonie choloeyetitis Answer: B- similar to question 1, anabolic steroids also produce intrahepatic cholestasis 33, A A2-yearold Asian man with posinecrotie necrosis secondary to chronic hepatitis B has a low ‘grade fever, weight loss, and a rapid development of ascites. A peritoneal tap reveals bloody ascitic ‘uid, The fluid WBC count is normal and bacteria are not present on Gram stain, The ‘acfetoprotein level is $00 ng/mL. (normal < 6 ngiml.). The MOST LIKELY diagnosis i. SELECT 1 ‘A printary gallbladder cancer 1B epatocellular earoinoma ©. metastatic Tver disease D. spontaneous peritonitis E. cholangiocarcinoma Answer: B~ this is the classic history for HCC. Note the increase in APP. 14. A patient living in a Basque community in Southem Arizona presents witha bistory of xecureit “right upper quadrant pain. Hl is a dheepherder and asa dog that helps him keep the sheep ‘ultrasound ot hs liver reveals a eystic mass in the live with caleifcations in the lining ofthe tyst, Which ofthe following additional findings would you expect in this patient? SELECT 1 A. His dogis an intermediate host B. He isadefinitive host CC. Heate an infeoted sheep D._Hiisdog ate an infected sheep ‘Answer: D~ the patient has echinococcosis, The cycle forthe parasite is egg to larva to sult. The host “with the larva is always the intermediate host (infected sheep, sheepherder), while the host with the adults is the definitive host (dog who ate the infecled sheep), The shcepherder pot the disease from ‘the dog who has the adults that produce eggs. The eggs develop into the larva in the sheepherder, \who isthe intermediate host, Basque's and Gieek’s ate fous: 15, A weightlifter develops «sudden onset of abdominal pain along with hypovolemie shock. At surgery, his absiominal cavity is filled up with blood. The canse ofthe itraabdominal bleed is most likely associated with which of the following? SELECT 2 A. Anabolic steroids B. Hepatocellular carcinoma C.__Liver cell adenoma 1D. Cavernous hemangioma in the'liver E. Straining at stool Answers: A, C- always assume that weighiliNers, wrestlers, or professional foatball players are on steroids, Anabolic steroids and estrogen not only produce intrabepatic cholestasis bat also produce benign tumors called liver cell adenomas. They have a tendency fo rupture and produce intraperitoneal hemorrhage, 16, A 3S-year-old executive ofan up and coming computer company develops a sudden anset of severe spigastric pain with radiation of the pain into the back. He deseribes the pain ws knife-tike, In dition, he feels nauseous and has vomited non-bile stained fluid on two occasions. Physical exam reveals tendemess in the epigastric area but no rebound tendemess. There is mild tender hepatomegaly. A flat plate of the abdomen reveals a dilated loop of small bowel in the let upper quadrant. An ultrasound reveals no stones in the galbladér and @ normal diameter of the common, bile duct. A stool guaiac is negative. Serum AST is higher than serum ALT. What test is most indicated in this patient? SELECT 1 ‘A. Upper gastrointestinal endoscopy B. Serumamylase and/or lipase CIDA (radionuclide) soan to R/O a eystic duct stone 40 [Notes This material i copyrighted, All rights reserved. (Edward F; Goljen, M.D.) 2002 D. Hepatitis serologies E, Surgical consult to R/O small bowel obstruction Auswer: B~ the patieat isan aloobolic (AST > ALT) with acute pancreatitis (pain radiating int back and sentine! loop due to localized ileus) ‘Renal questions 1, An 82-year-old man presents with lower back pain and complaints of problems with voiding urine, ‘There is point tendemess over the lower lumbar vertebra, Hix bladder is percussed at the level af ‘the umbilicus. Which of the following tests or procedures is indicated as the frst stop in the ‘management ofthis patient? A. Radionvelide bone sean B. Prostate specific antigen C. Digital rectal examination D. Serumalkealine phosphatase FE. Transtectal ultrasound with biopsy Answer: C~ the patient has metastatic prostate cancer. The best first testis a reetal, which would definitely identify cancer. The other tests can thea be ordered. 2. A 28-year-old:man with a history of removel ofa right cryptorchid testis asa cil is noted to have €& painless enlargement ofthe remaining testicle inthe left serotal sc. The testicle is enlarged and does not transilluminste. The patient most Hikely has... AL hytkocele B. seminoma ©. varicocele D. yolks tumor E._ choriocareinoma Answer: B- the key isthe eryporchidism. Even the uninvolved testiole is at risk, 3. A Go-year-old man with a 45 pack year history of smoking presents with hematuria, ever, and a ‘lpable mass in the left lower quadrant. A chest x-ray reveals multiple nodlat sasses in the Jungs. These findings most strongly suggest which ofthe following diagnoses? A. Miliary TB involving the kidneys B. Renal cell carcinoma with lung metastasis C. Primary tung cancer with metastasis to the kidney 1D. Choriocarcinoma with metastasis to lungs and kidneys E. __Acate pyelonephritis with metastatic abscesses inthe ing Answer: Bnote the smoking history, Roth lung and renal cancer ate aseovinted with amioking, In this sate, hematuria and the Hank miass indicates the kidney as the source. Renal metastasis is “uncommon, 4. A febrile 23-year-old woman presents with an acute onset of right flank pain, suprapubic iscomfor, dysutia, and increased frequency of urination. ‘The urinary sedinsent examination reveals clumps of leukooytes, WBC casts, occasional RECs, and numerous motile bacteria, ‘The ‘mechanism of this patient's urinary condition is most closely related to... A. arena stone B. ascending infeetion ©. immunocomplex disease D. drug-induced interstitial nephritis E, _ hematogenous epread of infection othe kidneys Answer: B~ the patent has acute pyelonephritis (fever, WBC casts, lank pain) This is due to ascending infection and vesicoureteral refx 5. A 25eyearold male presents with a history of hemoptysis and malaise, His blood pressure is 140/90 mun Hig. He has periorbital edema and smoky colored urine. Pertinent laboratory data 4a ‘Note; This material is copyrighted, All rights reserved. (Edward F. Goljan, M.D.) 2002 ‘include a scrum urea nitrogen of 40 mgidL (7-18 mp/dL.) and a creatinine of &0 mg/al. (0.6~1.2 mg/d). Usnalysis shows 2+ protein, 3+ blood, RBCs to0 numerous fo count, and RBC casts. A ‘chest radiograph: demonstrates opacities in both lung fields. The mechanism of this patient's lung ‘and real disease is most closely related toa ‘A. ypeThypersensitvity reaction B. type Ihypersenstivity reaction ype Il hypersensitivity reaction D. _typelV hypersensitivity reaction B_ type Hand Il hypersensitivity reaction Auswer B- the patient has Goodpasture’s syndrome. Note how patients usually start the disease in the lungs. and thea. the renal disease occurs. Anti-glomerular and puliionary capillary basement ‘membrane antibodies are responsible, Rapisly progressive erescentic GN isthe most common type of renal disease. Thoms 6-7 ©. Focal segmental glomensiscerais D. Rapidly propsesive cresceatic glomerulonepbits E. _TypeTmemtrmopeliatve glomerlanephrts ype 6 A 7A-yoarold man with # 80 pack histo of cigarette smoking presenls with Kemopiysis and Shoriness of breath Radiographs of the chest demonstrate left hilar mas. Heals has generalized nataren with 4 proscinria, hypercholestrolemia, and sty cass in the urine, ‘Ansuer:B- association of Tung cancer with membranous GN, whch isthe MC type of GN easing the ‘nephrotic in aduls 7. -A'Myearold male who is a known drug secker and heroin addict oomes to the emergency department inan agitated slate, He is restrained and you note multiple needle tracks. on both arms. Tre aio as eovere piling edema of Ue lower extremities, Untalysis is positive for protein and shows fatty casts on urine cement, Answer: C- fed! segmental slomerilsciross isthe MC ten disease in 1VDA and AIDS. Iisa severe ‘ype of nephrotie syndrome. 8. A'T0.yearold-adolescent boy peesents with unilsteral, fender suherenler mas inthe left breast. Psical exam is olherwise outennskable. Which ofthe following would you recommend for his patient? ‘A. Serum gonadotropins BL Chromosome analysis © SerumBhCS BD. Surgical eons E._Noleatneat “Answer: E~pubetal gynecomastia. Resll that eynecomasta is normal at bth, puberty, end in old spe, relates to hyperestrinism. 9. A2syearold child presents with dyspnea and hypertension. Physical exam reveals. palpable right lower quadkant mass in the abdormen and sctired sibilant shonchi in the Hons. A chest x7ay reveals multiple nodular masses inthe lungs. These findings most strongly suggest which ofthe following diagnoses? A. Wilm's tumor B. Neuroblastoma ©. Carcinoid tumor D. Renaladenocartinona E, _Dysplastc kidney disease ‘Answer: A-note the history of hypertension and a unilateral abdominal mass a 'Note: This material is copyrighted. All rights reserved. (Fdward F, Goljan, M.D.) 2002 10. A normotensive &yr-old boy, with 2 history of an upper respiratory infection 1 week ago, now ‘presents with generalized pitting edema. The urinalysis reveals severe proteinuria, fatty casts, and ‘oval fat bodies. fis condition rapidly improves with high doses of corticosteroids. Which of the following glomerular changes would you expect inthis patient? A. Fusion of the podaaytes B. Diffuse proiferaive changes C. _Diffvse membranous changes D. — Subepiieial immune deposits B, _Subendothetal immune deposits ‘Answer: A~ the patient bas lipoid nephrosis IL ‘A T2eyeold man presents with hypovolemic shock secondary to a ruptured abdominal sovtc. ‘peuryam. Within 24 hrs of surgery, he develops oliguria. He died 3 days Inter. Which of the following laboratory abnormalities is most likely presen this patent? A. Wie blood col casts Bh. BUN¢reatinine ratio > 15/1 Urine Osmolality > 800:mOsmvig 1D. Fractional excretion of sodium 1 R. Random rie sodium <20 mi. Answer: D~ the patient hes ischemic ATN, BUN/reatinine ratio should be 40 mig (tubular dysfunction) A 32-yrold man with hypertension has a eystolie elick followed by a murmur. Hie has a family Distory of hypertension at un carly age often progressing into chronic renal failure requiring either dialysis or a renal transplant. A few of the family members have died from a stoke related to an intracerebral bleed or subarachnoid hemorrhage. Te patient most likely has, ‘A. adultpolyeyatic kidney disease B._renal artery stenosis © renal éyaplasin D. IgA glomersionepbritis E. diabetic nephropathy Answer: A~ APKD is an AD disorder, hence the family history, Mitral valve prolapse is » common 1B. association including intracerebral bleeds and subarachnoid hemorehages from ruptured berry angurysmns. ‘A.32-yr-old man with Wegeners granulomatosis has hematuria, A urine eytology reveals numerous RBCs and opcasional atypical transitional cells, Which of the following lest explains the renal problems jn this patient? A. Renal disease astocited with Wepener’sgranulomstosis B. Chemotherapy agent the patient is taking to reat hi disease ©. Renal adenocareinoone imrelated to his disease D. Adenovirus infection E. Lower urinary tract infection Answer: C- cyclophosphamide is the Rx of choice for WG. It produces both hemorshayic cystitis and ¥ ‘transitional cell carcinoma, Atal, obese 13-year-old boy has bilateral gynecomastia, deceased testicular yolumné forage, and sparse axillary and pubic hait. He has had the usual childhood infections except for mumps. A CT. sean of the sella turoic js normal, A chromosome study is pending. Based on the pisieal findings and CT sean results, which of the following serum follicle stimulating hormone (FSI), seam luteinizing honinone (LH), and serum testosterone levels is expected in this patient? Serum FSH — SerumLH Serum ¢estosterone Chromosomes, A High ‘Normal ‘Normal 46 B, Normal High Tow 46 Cc igh High Low a a ‘Note: This material is copyrighted. All rights reserved. (Edward F. Goljan, M.D.) 2002 D. Normal Normal Normal 46 EB Low Low Low 46 ‘Answer: C- this is Klineflte’s syndrome. A~ pure seminiferous tubule failure, B= pure Leydig cell failure, D = normal person or testicular feminization if the elinioal history was correct, E = ‘combined seminiferous and Leydig cel failure Gynecology questions ‘eras 14 A. Inflating ductal carcinoma B. Lobular carcinoma ©. Fibrocystie change D, Intraductal papilloma E. Intraductal carcinoma Paget's disease 1. A 28-year-old woman during her breast exam is noted to have a bloody discharge. No masses are palpsble. ‘Answer: D~ these develop in the laciferous duct, 2. A 65-year-old woman with a history of breast cancer in her mother is noted to have retraction of skin in the right upper outer quadrant when abdueting her tight am doring her ante! 3/1, serum total testosterone: slightly increased, serum ‘ge testosterone increased, serum DHEA-eulfate noreal, serum prolactin normal, and the sorum ‘TSH{is momma. Based on these findings you strongly suspect thatthe patient has. A. bilateral androgen-seeretng tumors of the overies B. deficiency of gonadotropiercleasing hormone CC. _polyeystic ovarian syndrome D. adrenal Cushing's syndrome E. _ a gonadotroin-scerting pituitary tumor Ausirer: C~this is as classic a presentation ast gots 14, Which of the following patients sat greatest risk for developing endometrial cancer’? Present Age Menarche Menopause Children Miscellaneous A 45 1 e none Hypertension B. 54 2 st 2 Obese, took birth ‘contr pills © 56 2 30 none ‘Type diabetes mellitus D. 58 "1 3 none Obese, diabetes amelie BR 55 u 50 none Obese, took birth ‘contr! pills “Answer: D~ early menarche, late menopause, niliparity, obesity, type TDM are all increased risks for cadometil cancer, which relates to prolonged exposure to Uaoppo%ed estrogen 46 "Note: This material is copyrighted. All rights reserved. (Haward F. Goljan, M.D.) 2002 15. Inwhich of the following clinical situations, does the patient have the greatest risk for developing cervical canoer? ‘Onset of sexual Number of Eresntage activity, sexalpariners Mscllaneons history ‘A; 20years old 14 years old ‘Obese, on bicth eontrl pills BR. 19yearsold 10 years old a On birth contral pills, smoker © 19Years old 12 years old 3 ‘On irth contol pills, obese, smoker D. 21 yearsold 10 years old 10 Intravenous drug abuser, smoker, on bith control pills E, 21yearsold 10years old a ‘Obese, smoker, current boy- ‘iead ie an intravenous drug abuser Answer: Dearly sexual exposure, multiple high risk sex partners, smoking, birth contol pills sre tisk factors 16, A257 woman rests with ahs fever calc panenates, pif toting dng ‘nense, an problems with infertility. Laparoscopy reveals scarring around beth llopian tubes, 3 ‘om oyt in te right ovary, end "powder burn’ appearing areas on the uterine ligaments and serosal surface of the wes. The mechanism for this patents constellation of probiems is most likely rebated. ‘A spread of endometrial carcinoma 1B. _ seeding fom a primary ovarian tumor C. reverse menses through the flops tubes D. complications related io pelvic inflammatory disease E. _ endometrial glands end stoma in the myometrial tissue “Answer: C~ the patent has classi endometosis 17, A shortstatured 16-year-old gil resets witha history of primary amenortea. Piysical exam reveals poor breast development and sparse axillary and pubic ir. Hr an i redundant at the base of the neck. Pelvic exam exhibits a small utes and a solid eR ovarian mass. She bas no ‘Withdrawal bleeding aftr a progesterone challenge. Which ofthe fllowing laboratory et findings ‘would you expect in his patent? ‘Serum ISH SerumLH — Serumestradiol_ Chromosomes A Bligh "Nocmal Low 45 B. Norm High Low 46 © Tigh High Low 45 D. Normal ‘Normal Noa 46 E Low Low 45 FSH fli eating onmans, LA = ting home Answer: C- the patent iss Tuer’ eyndrome, the MC genetic cause of pritnary amenorrhea. Endocrine questions ems 1-3 rs 7S zy ‘A [nereased | Decreased | Tnereased B._| Inereused_| Normal | Not indicated (C_ | Inereased | Decreased | Decreased D. E i Decreased! | Notmal | Not indicted Decreased | Inereased | Not indioated ‘A-45-year-old obese woman presents witha history of weight loss, weekness, and palpitations that Jeep her awake at night, She states that she has lost over 50 pounds the last & months while 47 ‘Noto: This material is copyrighted. Allrights reserved. (Edward F. Goljan, M:D.) 2002 attending 2 weight loes clinic. Physical exam reveals i stare, a non-palpable thyroid gland, brisk deep tendon reflexes, ystlic hypertension, and an ire gularly irregular pulse. She isnot taking any prescription medications other than the packet of pils given to her by the cin. Answer! C- patient js faking exoes thyroid hormone, Note the Tow Lik would be high in Graves disease 2. A 22-year-old woman complains of intermittent Matering in her chest, Physical exam reveals ‘normal thyroid, no Tid stare or exophihalmos, a regular heat rate of 108 beatsminutc, normal deep fendon reflexes, anid blood pressure of 100/80 mm He. A mid-systolic cick and murmur is heard at the apex that incceases with expiration. She is currently taking birth contro pills. Answer: B- estragen effect on inereasing TBG without altering the free hormone level. Mitral valve ‘prolapse isan incidental finding 3. A 26-year-old woman presents with complaints of chronic constipation and progressive weight grin ‘over the last 6 months in spite of being on.a pure vegan det. She is curently on no preseiption or fovershe-sounter medications. Physical cxam exhibits « pale young: woman vith periorbital puifiness, dry, yellow-coloced skin, normal scl, a normal cardiovascular and respretory exam, ‘Selayed deep tendon reflexes, and proximal muscle weakness i het lower extremities, Answer: E~ pinay hypothyroidism A = Graves disease, D = Androgen effect. ems 4-5 Boo rH z= aD ‘Serum Calcium Note he square repreceats normal values 4, A.SS-year-old woman with diastolic hypertension pregona with x sudden onset of ht nk pain ‘with radiation of pain ato the right groin. A ucinelyss reveals a positive dipstick for blood and 2 “nopative dipstick for nitrite and leukocyte esterase, Sediment exam exhibits munezous RBCs and square ental resembling the back of an envelope. “Answer: rena stone in primary HPT 5." Annewhom presents with tetany, heart fllure, and an absent thyme shadow. “Ansiver: A~ DiGeorge syndrome, which has absence of the thymus and parathyroid glands ftom failure ‘of development of the 3rd and 4th pharynges! pouches. Choiee A eould also be primary Typonarathyroidiam. B= secondary hyperparatiyroidism, D = malignancy induced and al oher types of hypercalcemia . 6 Which ofthe following cinical and laboratory abnormalities occur in BOTH Addison's disease and pashypopitutaism? AAO" Tow Lt-deoxycortsal post-metyrapane test B. Norm urine for 17-Aetosteroids CHypematremia ‘D. Hyperkalemia E LowTSH Answer: A~ metyrapone blocks 11 hydroxylase, so ACTH and L1-deoxyeortisol should be increased if the pituitary and adrenal cortex are normal. In hypopituitarism, neither ACTH or deoxycortisol is ‘increased. in Addison's disease, ACTH is increased, but deoxycortisol is decreased. Choices B and Covour in neither condition. The 17 ketosterids (DHEA. end androstenedione) would be low in ‘oth of them. Both would have hyponatremia. In hypopituitarism, the loss of cortisol and thyroid hormone leaves ADH unchecked, therefore there is a mild hyponatremia. Remember that ACTH bas no effect on aldosterone release. In Addison's, there is a loss of aldosterone, hence 48 ‘Note: This material is copyrighted, All ights reserved. (Rdward F. Goljan, M.D.) 2002 hyponatremia is much more severe. Choice D is only present in Addison's and choice F is only ‘present in panhypopitutarsm. ‘Which of the following, alterations in lipid metabolism is expected in a patient with diabetic ketoacidosis? A. Activation of capillary lipoprotein lipase 1B. Increased B-oxidation of fatty acids C. Decreased production of acetyl-CoA. D. Increased fatty acid synthesis 1K, Inhibition of hormone sensitive lipase ‘Answer: B- when insulin is absent, fatty acids tmdergo B-oxidation. Choice A. occurs in the presence of ‘insulin ond is inhibited in DKA. Choice C occurs when insulin is present in onder to synthesize fatty acids or when it is absent (DKA) due to increased B-oxidation of FAs, Choice D oceurs when insulin is present and js inhibited in DKA. Choice F occurs whea insulin is present. The enzyme is activated in DKA due to glucagon and gatecholamnines. 8. A 22-year-old woman hay been amenoreic for the last 6 months, She complains of w silky from her nipples. that has been present for the last’? months. The pregnancy test ‘negative and the serum TSH is normal. Which of the following additional teats ehould you order on this patient? A.” Serum prolactin B,-Metyrapone test © SenumT, D. Serum cortisol B. Serum gonadotropins Answer: A~ the patient has a prolactinoms, Prolactin inhibits GnRH, hence the amenorshea. Since the ‘TSH ia normal, primary hypothyroidism eaniot be the cause. 9. Which of the following tests distinguishes pituitary Cushings from both adrenal and ectopic Cushings? A. 24-hour urine for 174etosteroids B. Lowdose dexamethasone test © Serum cortisol level D. High dose dexamethasone suppression test E. 24-hour urine for free cortisol Answer: D~ i pituitary Cushing's, coriolis suppressed withthe high dose dexamethasoie test and not in other types of Cushings. All the other choices are the same inal of them. 10. A 35-year-old pharmacist presents to your office wis reeutrent episodes of forgetfulness and tiredness. A. serum glucose is reported to be 20 mg/dl. (70-110 mgla.). Additional studies on the Same sample reveal a high serum insulin and high C-pepide level. Based on these findings, you ‘suspect the patient hs... ‘A. abenign tumor involving B-islet ells B. aneatly phase of type mllitus ©. a benign tumor involving islet cells D. ectopic secretion ofan inslinlke factor 1B, _ surreptitiously mjectediuman insulin Answer: A~ the hieh C-peptide proves thatthe patient has an insulinoms. It would be suppressed ifthe patient was sureptitiosly taking. buinan insulin, sinee it would suppress endogenous insulin release, 11, A 39-year-old type Tinsulin dependent diahetc has » burning sensation around his ankles and on the boroms of both feet. Neurological examination reveals depressed Achilles and knce jerk reflexes bilaterally and deereased light touch sensation in both lower extremities, The mechanism for this is most closely asociated with, A. osmotic damage 4“ Note: This material is copyrighted. Allright reserved. (Edward F. Goljan, MD.) 2002 Uhizesine deficiency pernicious anemia syringomyelia lumbar disk disease Answer: A- osmotic damage of Schwann cells produces a sensorimotor peripheral aeuropaty. Glucose 4s converted by aldose reductase into orbital, which is osmotically ative. 12. Which of the following water deprivation test results would you expect in a patient with polyuria groab after head trauma’? ‘POsm post. UOsm post WOsin post water waterdeprivation vasopressin deprivation normal (29. 750 760 A 312 98 120 B 319 10 350 & 288 760 780 ‘Answer: B~ in patica’ wit either central or nephrogenic Dl, water deprivation results in alow UOsm ‘ADH casiot concentrate the urine) anda high POsm, which carelates with hypernatremia. When ‘vasopressin is given tothe patient, those with central DI will havea rise in UOsm >50% from the baseline, while those with nephrogenic DI have less of a response (choice A). Patients with psychogenic polydipsia behave like a normal contro, in that they ean eoncentate urine with water vation. "3, Whi fie flowing most ley dictates pe om ype Gees linet A. Insalitis B.Pesitve family istry ©. Hypoglycemic reactions 1D. Posteinsulin receptor defects B. _Increased glycosylated hemoglobin Answer: A~ inslitis is due to autoimmune destruction of the feels. In type If DM, the islets are fibrosed and ofien contain amyloid. Choices B and D are associated with type II DM. Choices C and F occur in both types 14, An 8.year-old boy presents with headache and bilatcral hemianopsi. A CT scan of the skill evesls enlargement of the sella turcica secondary toa cystic mass with multiple caleifications. Which of the fllowing isan expected borin ding in his pent? ‘Low insulin grow factor-I levels “Hyponatremia and hypeckalensia ‘Normal urine for 17-hydroxyeorticoids High serum TSH and low serum Ts "Normal serum 1 -deoxyeortisol after metyrapone ‘Answer: A- the child hs a craniopharyngioma with optic chiaem compression and hypopituitarism. IGF- 1 would be low. Hypokalemia, low urine 17-0H levels (cortisnl and deaxyeortisol), low serum ‘TSH, and low serum deoxyvortsol aftr metyrapone would be expected. 15. Adf-year-old man with diastolic hypertension complains of headaches, drenching night sweats, and palpitations that keep him awake at night. He has a family hibtory of hypertension. Physical ‘exam findings include an anxious man with a mean blood pressure of 152/96 mm Hg and a regular pulse of 148 heats/minute, There is no evidence of thyeomegaly. Deep tendon reflexes are brisk. ‘Which ofthe following tests is the best screening test fortis patient? ‘A. Serum thyroid stimulating hormone 1B. 24dnr urine for metanepines ©. 24h rine for free cortisol D. B Hope Plasma aldosterone Metyrapone test 0 ‘Note: This material is copyrighted. All rights reserved. (Edward F. Goa, M.D.) 2002 Answer: B the patient has pheochromocytoma, The history i classi. Metanophrinos are more sensitive ‘dian VMA asa screen, 16, A2-yearold child with newly diseayered diastolic hypertension has multiple modular lesions on the skin. A biopsy of one of th lesions exhibits an intimate of srall, round, hyperchromatie cells. The cells stsin positive for $100 antigen, A radiomelide bone scan is reported to have multiple Iucencies vonsisent with metastatic disease. The primary origin of the patients skin and marrow infiltrate is from which ofthe following sites? A Skin B. Bone © Kigney: D. Thyroid E, Adrenal cortex Adrenal medalla ‘Answer: F- the child has'a neuroblastoma with metistesis. Neuoblistomas are APUD tumors. They ‘of the follewing biochemical reactions is most responsible for hyperglycemia in a patent with diabetic ketoacidosis? A B, a D. py Pe glucose E. Citrate ——— Oraloscetate > Malate po glucaso+ Acetyl Co DBAP = dikydroxyacetone phosphate ‘Answer: D glaconeogenesis is the key reaction that produces hyperglyeemin in DKA, dhetefore the alain to pyruvate to glucose isthe best answer 18, A26-y-old woman with an abruptio placenta develops hypovolemie shock requiring multiple units ‘of packed red blood cells. Her baby is delivered without consequence, Two daye later, the mother ‘experiences a sudden cessation of lactation. The pathologic process most likely responsible for this complication i. A. hemorrhage into a preexisting ptitary adenoma 'B._inoreased secretion of dopamine ©. coagulation necrosis. ofthe anterior pituitary D. benign pititary adenoma with secrecion of prolactin E, _ hypothslamie dysfunction leading to hypopstiarsns ‘Answer: C- the patient has Sheets postparium necrosis. Prolactin is the first hormone to decrease, Thence eessation of action. 19. A 22-yrold type I diabetic devclops sweating, anxiety, excessive hunger, and light headeiness. “The first step in the management ofthis patent iso. A order plats Be B. orders serum insulin and C-peptide level ‘Corder sum electrolytes D. ordera:24 hr rine for metanephrines F._ give the patient candy bar ‘Answer: E= these ae the symptoms of hypoglycemia from too much insulin. A candy bar will suffice, 20, A 25-yr-old man with non-tender thyromesaly and palpable cervical Imps nodes has an elevated serum calcitonin level. A fine needle aspiration of the thyroid is scheduled. He has a family history of thyroid disease, hypertension, and renal stones. Which of the following additional laboratory tests should be orderet on this patient? ‘A. Serum electolytes and 24 br urine for free cortisol B. 24 hrurine for metanephrine and serum PTEVealcium 31 ‘Note: This material is copyrighted Al rights reserved. (Edward P. Golian, M.D.) 2002 ©. Serum aldosterone and 24 urine for potassium D. Serum gastrin and gastric analysis to measure hasel ac ontput B. _ Scrum vasointestinal peptide and somatostatin levels ‘ers tvs A soe Serve: alone $e tec ete ‘hyperparathyroidisma, and pheochromocytoma. 21. A 2L-yrold man with a history of infertility is noted to have a Large varicocele inthe lef serotal ‘si. His physician suspects a problem withthe seminiferous tubules. Which ofthe following sets of Inboratory test results would you expect inthis patient? Serum Serum — Serum FSH LH testosterone Sperm count ‘A. High Normal Norma Deoreased B. Nowal High Low Deoreased © High High Low Decreased ‘D. Normal Normal Normal Normal Answer: A~ since the seminiferous tubules have Sertoli cells that synthesize inhibin, which is the ‘negative feedback for FSH, the PSH would be increased. Leydig calls synthesize testosterone, “which has a nopative feodback with LH, s both of these test would be norma. Tbe speren counts low, since they develop in the seminiferous tubules. Choice B is Leydig eel dysfunotion, choice C {s bath seminiferous and Ley ell dysfunction, 22 A 45-year-old man with diastolic hypertension presents with muscle wedkness and telany. ‘Laboratory stidies reveal mild hypernatremia, hypokaleni, and metabotic afkalesis. The is mo Syidene of iting edama or ole depletion. The patent mos kay bas B. ehvonic renal failure C.prmary aldosteronism ‘D, been taking 2 Joop diretic E _ type renal tubular acidosis. Answer: C~ primary uldosteronism. Aside from Cushing's syndrome, no other disorder has this type of ‘leettolyte profile + hypertension. Tetany is doe to metabolic alkslosis. An alkaline pH increases ‘the number of negative charges un albumia, hence more ofthe ftee ionized calcium can be bound to albumin. The total calcium remains the seme, but the ionized calcium is decreased, hence the presence of teeny, ‘Musculoskeletal and autotmumine questions 1. A sexually active 30-year-old man presents with fever and pain i bis sight big toe that woke him up at night He hs been taking aspirin to relieve the pain without relief. His mothor has severe osteoarthritis. The right toe is ewollen, hot and exquisitcly sensitive to touch, Laboratory studies seveal a neutrophilic leukocytosis and left shift. A synovial tap was performed. Based on the above history, his problem is most likely related to... ‘A. joint inflammation secondary to 2 positively biretingent crystal B. disseminated gonococvemia with septic arthritis C.—underexcretion of uric avid inthe urine 1D. _ osteomyelitis secondary to hematogenous spread of Staphylococeus aureus E, anlILA.B27 positive spondylosrtiropathy Answer: C- the patient has gout involving the big toe (podagra). It is most commonly due to ‘underexeretion rather than overproduction of usie acid, The crystals are negatively birelFingent, smconing that they are yellow when parallel to the slow ray ofthe compensator, 2. Which ofthe following characterizes joint disease associated with rheumatoid arthritis rather then osteoarthritis? A. Cartilage fibrillation 2 "Note: This material is copyrighted. All rights reserved. (Edward F. Goljan, M.D.) 2002 Suibchondral bone ey Osteophytes Ankyloss of the joint ._Bouchari!s nodes Answer: join fusion due to pannus destrection of the articular certlage is a characteristic of RA. OA {snot An inflammatory arthritis. All he other choices are findings in OA. 3. A 28:yearold man who works ina summer camp in upstate New York presents with blatrl fecal ‘weakness involving both the upper and lower facial muscles. He also complains of fint pains in both knees. He has a history a few weeks ago of having kad a peculiar rash on his right thigh that ‘was cioular, red eolored and had a eental area of elearing. The patients disease is most Kikely poos caused bya. ‘A. pramnogative diplococous B. spicochete ©. gram postive coves DB. vins E. _ immune complex reaction ‘Answer: B- the patent has classic Lyme dicase, due to the spirochete Horrelta burgdorfer). It is transmitted by the Ics tsk The rash fs erptheinachronicum migrans, Rx is doxyeyeline. 4. A febrile, sexually active 23-year-old woman presents with aot, swollen right knee and pustular lesions on the palm of her left hard. She recently returned fom a camping trip in Colorado. A gram sfain of the synovial uid in the knee and srom an aspirate of one ofthe pustlesrevesled a... A. grampositive diplococeus, 1B. gramnegative rod C.spiroctete 1D, _ gram negative diplocoocus , _nirmuelear inclusion Answer: D- ihe patient has disseminated gonococcemia due to Neisseria gonorrhoeae, a gram negative iplococeus. Some ofthese patients are deficiency in complement components C5-CS, ‘S.A veterinarian develops tenosynovitis within 4-hes ofan animal bite. You suspect the offending ‘organism is... AL Yersinia pests BL Eikenella corrodens © Fusobacterium D. Pasteurella multocida E. Staphylococcus aureus Answer: Dcat bites are mote likely to cause this than dog bites, Penicillin is the Rx of choice 6. Overgrowth of synovial tissue Autoantibodies against immunoglobulins Answer: C- sacroilitis and male dominance (HLA B27 positive) characterize AS. Pannus and ‘heumatoid factor (IgM anfibodies against IG) characterize RA 7. A 36-year-old man, who hes had problems with his lower hack since his early twenties, presents with blurry vision in bis riaht eye. Physical exam reveals a ciliary Hush and a poor pupillary ‘response to light inthe right eye. The intraoeular pressure is normal, He also has reduced anterior flexion and dry inspiratory crackles at bath lang bases. The patient is presently taking indomethacin for back pain, You would expect an x-ray of te patient's back to reveal... ‘A. radiodense lesions in the vertebra A B C.Ankylosis of the joint D. BE 3 ‘Note: This material is copyrighted All rights reserved. (Edward F. Goljan, MD) 2002 2 C. vertebral compression facies D. B. fusion of the vertebra _Ansrer:E-the patient has ankylosing pondyfitis and weit "A 28-yearokd man hax ten year History of interment left ower qoadtant pain associated with bloody dames. He now eonsplans of syzametresl joint pains in the knees, wrist, and ankles, as ‘well asin the lower back. Which ofthe following i the inciting agent most ely responsible far his cure dhumatologc problems? ‘AL Overproduction of ure aid B. Invasive gastroenteritis ©. Colorectal cancer D. Ulcerative colitis B. Groh’ discase “Answer: D- the patent has nlerative clits which in an HLA B27 positive individual ith trigger for “produsing ankylosing spondylitis. : 9 ‘A eyearold git has an aeate onset of spiking fover, rash, generalized pant lymphadenopathy, 0d polyaris. The cardiac eam is nome. A serum wiinelear antibody test, cheumatoid factor, and urinalysis are roportad as negative. A CBC reveals an ubsolu snevopilic lenkocyosi,sild norwoeyic anemin, and a normal platelet count. Which of the following best characterizes the mature ofthis patients disease? ‘Non-dsabling, noninfectious inflammatory joint disease Progrssively disabling infectious joint disease duc to bacteria Progressvely disabling nom-infectioas inflammatory joint disease ‘Non-diaabling infooious joint dicase du to an immunologic reaction against a bacteria ‘Progressively disabling noninfectious inflammetory joint deeace associated wih an HLA~ Barhaplotype Answer: O~ the paicnt has javenilevicumstold aris, poxifcally Sts disease. 10, Which of the folowing elinioopahotogic findings occurs in both osteoarthis end rheumari satis? A. Bhuny vision 1B. Panmus formation ©. Narow joint space D. Subchondral bone eysts FE. _Metacarpophalangeal joint involvement Answer! C- both los articular eartilage, bat for different teasns. In OA its wom down by weight and fschemia, while in RA, it's Gesttoyed by pannus. Only RA affects the MCP joint, Subchondral ‘bons eysis area feature of OA. 11, 45-year-old man has chronic myelogenous leukemia that hes progressed into an ace blast risis, eis curt being treated with unuliple atileukeznic agents. ne week into therapy, he develops liguri renal fue. Analysis reeals an acid pH and numerous crystals. Which of the following best explains the mechanism for this patiea’ cual faluce? A. Drug nephrotoxicity 1B. Overproduction of ro acid €. Leukemicinilration of the kidneys D. Underaotivity of 5 phosphoribosyl-|-pyrophosphate (PRP) E. _ Oversctivity of hypoxanthine guanine phosphacbosylvansferase (IGPRT) “Answer: B~ ineteased destruction of els leads to increased metabolism of purines into ure seid leading {o uate nepiopathy. The patent should have been given allopurinol 12, A 6Syearold man enlew the caamining room with broad-based ataxia, Positive newologial findings inchide a positive Romberg test, los of pain and vibratory sensation in the lower 34 ryARP Note: ‘This material is eopyrighted, All rights reserved, (Edward F. Goljan,M.D.) 2002 ‘extremities and alent deep tendon reflexes in the lower extremities. Both pupils acsommodate but do not constict with direct light stimulation. The patient's right knee is swollen and soft tissue swelling is noted around the joint. An x-ray reveals extensive rosorpton of hone, Which of the following best explains the pathogenesis ofthis patient’ joint disease? A. Septic arthritis B. Loss ofpain sensation ©. Immmocomplex disease D. Upper motor neuron disease E. _ Cavitary lesion in the cerviesl spinal cord Answrer: B- the patient has tubes dorsalis and a neuropathic (Charcot) joint. The patient is unsble to 13, Answer: E- the sympiom/s 4, Answ 15, Answ 16. ‘sence pin, Disabling joint disease, nodular lesions inthe ling sssocisted wih dust borne diseases, xerostoma, and splenomegaly characterize theumatoiogic disease with Which one of the following laboratory abnormalities? ‘AL Positive serum antinuclear antibody witha rim patter B. Positive bund test on askin biopsy €. _Antiribonucleoprotein antibodies D. Antireeniromere anibodies 1, _IgMantibodies against eG ‘complex characterizes theumatoid arbrtis. It desoribes associations with ‘eumoconiosis (Caplan's syndrome), Sjogren's syndrome (dry eyes and dry mouth, gic to autoimmune destraction ofthe laecimal glands and minor salivay glands), and Felis syndrome (Gplenomegaly with neutropenia). A. 32,yearold pregnant woman, who is inder treatment for hypertension with hydralazine, develops an abmupt onset of a left-sided pleural effusion, photophobi, and joint pans in both of het hnands. Which of the following antibodies is most likely present inthis patent? A. Anti-double stranded DNA B. Anti-centromere © Anti-histone D. Anti-se1-70 E AntiSm fer: C-the patient has drug-induced lupus, with hydralazine asthe offending agent. Procanamide is another ug. A 24-year-old woman with previous history of moming stiffness of both hands presents with ‘dyspnea, neck vein distention with inspiration, and muffed heart sounds, Urinalysis reveals RBC ‘casts, hematuria, and mild to moderate proteinuria. A serum antinuclear antibody tet i positive and has after of 11280. Based on these findings, the patient most likely has, ‘A, mixed connective tissue disorder B. progressive systemic sclerosis ©. systemic lupus erythematosus D, _juvenilerheumatoid arthritis, E. adult heumatoid arthritis jer: C- the patent hae SIL with a classi pericardial effusion. ‘Which of the following elinicopathologic findings is present in both systemic lupus erythematoeus and Sjogren's syndrome? Selerodactyly Positive syphilis serology Anti-SS-A (Re) antibodies Non-infectious endocarditis Dysphagia for solids and liquids FoR ss [Notes This material is copyzighted. All rights reserved. (Edward P. Goljan, M.D.) 2002 Answer: C- selerodactyy and dysphagia for solidslliquids is scen in PSS and CREST syndrome. A false ‘positive eyphils cerology due to ant-candiolipin antibodies is seen in SLE as wel as Libman-Sackss endourditi. ‘Skin questions 1. A Stiyear-old man with congestive heart failure develops cough and swelling ini the deep subeulaneous tissue. He is most likely taking... A. athiacide diuretic -B an ACE inhibitor ©. aphenothiazine D. digitalis E. _ acaloiumchannel blocker ‘Answer: B- ACK is also involved in the metabolism of bradykinin. Hence, an ACE inhibitor leads to an increase in bradykinin (inereases vessel pecmeability), whieh isthe mechanism for angiodenia and cough. 2, "The most common fungal cause oftinea capitis witha negative Wood!'s lamp is... ‘A. Trichophyton tonsurans 1B, Aarosporum canis ©. Trichophyton ruiram D. Gandida albicans B Malassezia furfier ‘Answer: A- T- fonsurans invades the inner hair shaft, henge the fluorescent metabolites cannot be "detected by Wood's lap ligt (UVA light). It ie now the MCC of tinea expits. A. eanis sed to be the MC cause of tinea capitis. It is Woods lamp positive. 3. A43-yearold woman presents with joint pains and a buttefly-tike rash on the face. She is most likely taking. AL thiazides Boral contraceptives ©. hydralazine D. — doxyeyetine E, barbiturates Answer: C- proctinamide is another drug that is associated with drug-induced SLE. Antt-histone antibodies are pathognomonie of drug indooed lupus, 4. ‘A-42-year-old man has recurrent development of vesicular and bullous lesions in sux-expased ‘areas, He has had to avoid aleobol, because it seems lo coincide with these episodes. You would pet hpi. ‘istry of sbdorsinal pain a lnwctonin ancl ld aM wa, . —aminrease in porphoblnogen in his urine D. colorless urine during these tacks B. _adeereas in red blot cell uroporphyrinogen decarboxylase “Answer: E- the patent has porphyria cutanea tarda, 5. A 30-year-old female with «long history of alcohol and barbiturate abuse presents wth diffuse colicky abdominal pain, Examination reveals numerous surgical scars on the abdomen. ‘There is no Nery of ptoestviy, Yu suc tater rine woul. ‘contain an increase in uroponphyrin contain an inerease in coproporphyrin be positive for blood have an excess amount of urobilinogen tum a port wine color after exposure to light 36 PpoRE ‘Note: This material is copyrighted. All rights reserved, (Hdward F, Goljan, M.D} 2002 ‘Answer: B~the patient has aout intermitent porphyria 6. A baschall player develops a vesioula rash around the 1eck and on his forearms. He is most Hkely {aking which ofthe following dcugs? A. Thiazide diuretic B. ACE inhibitor © Tetracycline D. Pencil Calum channel blocker Answer: O- note that the distribution of the rash isin sun-exposed tens, so a drug associated with photosensitivity is likely. 7. A 25-grold sexually active man has a-verrucoid appearing lesion on the shall of his penis, The infections agent that is responsible fr this penile lesion belongs to which ofthe following subtypes ‘of microbial pathogeus? A. Spirochete B Rickensia Chlanyaia D. Fongus Vins ‘Aasvers =the pstent as condyloma souminat, whick is doe to HPV 6 or 11. Topical podopbylin is ‘he Rx af choie. Dano confise tis with condyloma fatun, which i secondary syphilis, 8 A35-yi-old black man has sely lesfons on hs seep and raised, plaque-ike lesions on his elbow and trmnk. The lesions have silvercolred scales on the suciace. Which of the following is & ‘common clinical finding associated with his ks lesion? A Positive Nikolsky sign B. Nailpiting © Dermatographism D. Wickham sia Squamous cel eacinoma ‘Answer: B~ the patient has psoriasis, In blacks, the eryema of the lesion is masked, however, the very seals are present. Nikolsky sign is noted in pemphigus vulgaris «bullous lesion with separation sbove the basal cell lnyer. Dermatogrephism is noted in chronic urtiari Wiekham's Stine fine, lay Teukoplakc lesion on the buccal mucosa in hen planus. 9. Which of the following most deicemines the prognosis of a superficial spreading. malignant relanoma? ‘A Gender of the patient BL Depth ofinvasion ‘©. Duration of exposure to sunlight D. Sizeasid color variation E. Presence ofaradial growth phase Answer: =the Breslow ystem measures doph of invasion. Lesion > 1.7mm in depts have the capacity to metasiasze. 10, An month old infant tas a weeping, eozemstnas rash onthe cheeks, The child is a mouth breather and constantly has a stufly nose and ofits media infections. Which ofthe following ebaractriaes the pathogenesis of this lesion? ‘A. Aulounibodies against DNA B. Type hypersensitivity restion C. Type hypersensitivity action D. TypelV hypersensitivity reaction Superficial dermatophyte infostion Answer: B~the patent has atopic dermatitis. Atopy i a characteristic feature in the family. 3 ‘Note: This muri is copyrighted. All rights reserved. (Edward F. Goljan, MD.) 2002 11, A.A-yrold woman has 2 scaly, eezematous skin lesion related tothe nickel in the metal clip on her {irdle. The pathogenesis ofthe skin lesion is similar tothe pathogenesis of... ‘A. wheal and flare reaction associated with a bee sting B. a maculopapular rash on a patient taking penicillin ©. palpable purpura in a small vessel vasculitis D. _ the'rashin poison ivy B. tinea corporis Answer: D- the patient has a typ 1V contact dermatitis to nickel, which is similar tothe pathogenesis of ‘poison ivy and a positive PPD. Choice A and B are a type reaction involving histamine. Choice C 4 type Il immunocomplex disease, Tinea corporis is a superficial dermatophyte infection, most ‘fen de to Trichqplgton rubrum. 12, 4 25-yrold woman is noted to have pruritic circular lesions on her Teg, They have a red, sealy border and a central area of clearing. Which of the following should be your fst, step in the diagnostic work-up ofthis patent? A. Punch biopsy B. KOH preparation Tanck preparation D, Peripheral blood count for eosinophils E. Urine sample for porphyrin analysis Answer: B- the patient has tinea corporis, which is most often due to Trichophyton rubra. 13. A Ayrold child has multiple eraterform lesions with "sandyslike" material in the area of ‘umbilication located over the trunk. Which ofthe following best describes the pathogenesis ofthese lesions? A. Viral infection B. Ultraviolet light © Fingal infection D. Bacterial infection E, Type hypersensitivity reaction ‘Answer: A~ the patient has molluscum contagiosum, which is a DNA poxvirus 14, A.Sieyrold farmer has raised, pearly gray lesions on the dorsum of his hands and forearms. Which ‘of the following best deseribes the pathogenesis of the pearly white lesion on the hand? A. Vira infection 1B. Ultraviolet light . Pung infeetion D. Bacterial iafeotion E, Type I hyperseustivity reaction Answer? B~ the patient has actinic (fol) keratosis, a UVB related precursor lesion for squamous cel, 15. A.25-yrold woman complains of multiple areas of skin that do not tan duving the eur. Paysical ‘extmination reveals areas of hyperpigmentation and hypopigmentation on her chest and back. A. KOH preparation taken from a the border of a hypopigmented skin lesion reveals yeasts. and liyphae. Which of the following skin disorder groups is sseocisted with the pathogen noted in the KOH preparation? ‘A. Tinea capitis: Pityriasis rosea B, _Sebortheic dermatitis: Tinea versicolor ©. Tinea corporis : Onychomycosis D. Pityriasis rosea : Tinea versicolor E, _ Onychomycosis; Seborrheic dermatitis ‘Answer: B~ the patient has tinea versicolor, which is due to Malassezia furfur, as superficial ‘dermatophyte that also exuses seborrheic dermatitis (dandrufi, 58 ‘Note: This material is copyrighted. All rights reserved. (Edward F, Goljan, M.D.) 2002 16. A 26-yr-old man piesents with slightly pruritic oval-shaped lesion on the tnmk that fae crythematous borders and a pale center. A few days later, a rath develops on the trunk that follows ‘the lines of Langer. The patent most likely has... ‘A. superficial dermatophytoses a vial exanthem pilyriasis rosea ‘an urticarial reaction contact dermatitis Answer: C- the intial rash is called a herald patch. The rash distribution is described as being in a “Christmas tre" distribution. Is best treated with UVB light. 17, A febrile Syr-old boy has # rash on the face, trunk, end extremities. Some of the lesions are flat, while others are vesicular, and others are pustular. A ‘Tzanck prep is performed and reveals ‘multinucleated squamous eels with intranxclear inclusions. The patient most likely has. A. an FISV-1 infection noes Bo varicella varia D. Herpes zoster rug reaction Answer: B- the patient has chickenpox, which is « DNA Herpesvirus in the same fury has HSV, CMV, and EBV. The inclusions of HSV and varicella look ths same, Note the different stages ofthe ash, ‘Which is charactoristio of waricela, 38, A 72yrold man with chronio lymphocytic leukemia has a painful, vesicular rast located on the shoulder. Ibbegins atthe midline of the neck and extends over the deltoid area, The pathogenesis of ‘the skin lesion in thie patient is ateibutabe to... ‘A, reactivation ofa Intent viral infeetion B. smallergic contact dermatitis © anulraviolet fight related dermatitis 1D. a group A streptooooens infection E. invasion of akin by leukemic celts ‘Answer: A~the patient has shingles. The virus i in the sensory ganglia in the thoracic area, 19. A 16-yr-old boy presents with yellow, crusted lesions aver the free, neck, and upper body, Ten slays afte the rash the patient develops periorbital puffiness, hypertension, oliguria, and mnoky finding in dhe patient is most elosely associated wit A. sepsis due to Streptococcus pneumoniae B, sepsis due mo Streptococcus pyogenes . sepsis duc to Staphylococcus aureur 1D, _animmunocompler disease associated with group A streptococcus E.__animmunocomplex disease associated with antibodies against DNA Answer: D- the patient had impetigo due to a ncplzitogenie stsin of group A skeptococeus which lead to post-streptococea!glomerulonepbritis. 20. 17-ye-old male bas facial lesions characterized by erythematous papular lesions with white and ‘lack centers. Some of the lesions have a nodular-eystic pattem. The pathogenesis of the inflammatory component of the facial lesions inthis patient is most closely associated with, ‘A. a group A streptococoal infection B. folliculitis secondary to Staphylococcus aureus ©. animmunocomplex disease associated with antibodies against DNA D. production of lipases by Propionibacterivm acnes E. _ folliculitis secondary to Streptococcus pyogenes Answer: D~ the patent has aone vulgaris 9 ‘Note: This material is copyrighted. All ights reserved. (Edward F. Goljan, M.D) 2002 CONS and special senses questions 1. Gapitvesbormais, ates, nyetagmas opto, an fot cop are al atcited ate einaey B. multiple sclerosis © Alzheimer’s disease D. alcoholism E._ Parkinson's disease Answer: Dall ofthe S/S are due to thiamine detiieney and Wernicke’s encephalopathy. Revall that the ‘manillary bodies and tise around the venivieles have an orange disooloration du to hemorrhages and subsequent deposition of hembsiderin, 2. A family history of chronic liver disease bepimning at an early uge and a movement disorder developing later in life characterizes a disease associated with... ‘A, atriplet repeat mitation Blow ceruloplasmin levels, C. thiamine deficienoy BD. exoess aloof intake E. _ vitamin Ba deficiency ‘Answer: B~ the patient has Wilson's disease, Degeneration oveurs inthe lenticular nuclei, It occurs in ‘he subthalamic nucles, hemibelismus may occur. 3. A dQyear-old_newropathologist developed @ rapidly progressive dementia and died. ‘The pathogenesis oFhis disorder is related to. A. decrease in acetyicholine levels B. deficiency of dopamine €. neuronal damage by amyloid D. alow virus disease involving prions E. _ subacute soleosing penencephalitis Answer: D~ the patent has Creutefeat-Takob disease 4. A 15-yeatold boy dovelops fever, nuchal rigidity, and petechial lesions. A spinal fluid reveals increased protein, deeteased glucose, inereased ncutropils, end a postive gram stain. You would ‘expect the CSF gram stain to reveal. A. grampostivediplocovei B. _ gramnegative coccobacili © gram positive rods D. sramnezative diplocoeei E. _ grampositive cocci Answer: D (Neisseria meningitidis). If the patient was over Iyer, the answer would be A (Gireptocgccus pneimoniae) 5. Which ofthe following represents a primary brain tunior that would more likely develop in acid rather than ap adult? ‘A. Glioblastoma mullforme B. Medalloblastema ©. Malignant iymphoma D. Acoustic neuroma E — Meningioma Answer: B- this is the MC primary malignant CNS tumor in children, but «cystic astrocytoma of the cerebellum (benign) isthe MC overall primary CNS tumor. 6 An afebrile 52-year-old smoker with weight loss, cough, and hemoptysis, develops a severe headache associated with unilateral lid lag, ophthalmoplegia, and mydvisis. An MRI reveals o "Note: ‘This material is copyrighted. All rights reserved. (Edward F. Goljan, M.D) 2002 ‘oltiple densities in the cerebel cortex. The pathogenesis of this patent's neurologic condition ost closely relates to... | i f cerebral edema with uncal herniation B.__ cerebellar heriation into the faramen magnum ‘Answer: D- the patient has metastatic km cancer tothe brain with nnealletiation and compression of the oculomotor nerve leading to mydriasis, 1id-ag, and ophthalmoplegia (eye: deviated dev wd ‘An astopsy is performed! on a 40-year-old man with dementia. Examination of the bein reveals atrophy ofthe font and temporoparietl lobes and senile plaques on hisologio examination ofthe ‘ssue, The pathogenesis of the CNS disease in this pateat is most closely rele to A. B-amloid protein 1B facuner infarcts ©. decreased dopamine levels D. slow virus disease E. _atipletrepeat disorder ‘Answer: A the patent has Down synrotne. The exta etiwmosotte 21 codes fo mote f-amyloid ‘role, hence Alzheimer’s disease at an early age 8 Recurrent episodes of blurry vision, scanning speech, and paresthesia is’ most closely associated © slow virus disease D. Huntington's disease E. Wilson's disease ‘Answer: A MS is the MC demyelinating disease, I isan autaimmune destrction of the myelin sheath. Recall that bilateral internclear opithalmoplega is pathognomonic of MS. "9, An encapsulated mass is removed ftom the right ecrebellopontine ale of 32-year-old mle with ehtsided senorineural hearing loss and facial numnes te sation of te an nerve, The patient most Hikely hts wai... A. ependymoma B. medulloblastoma ©. demyelinating disease D. meningioma E, acoustic neuroma “Answer: Z- sensory changes are due to VIlth nerve tumor in eerebllopontine angle imiiging on the ‘Vuh CN. There is an association of weoustic neuromas with neurofibeomatosis, 10. A 4S year-old woman with «history of mitral stenosis and cbronio arya died sudenty at hhome. At sutopsy, a hemorthagic lesion was noted at the periphery of the temporal lobe, Yhe patient most likely has wan... A. atheroselerote stroke 1B. intracerebral hematoma © embolie stroke D. glioblastoma raultiforme E. AV malformation ‘Answer: C- atrial fibrillaion isthe MC arshythmia leading 1o embolic discese. This ‘nfaretion liquefaotive necrosis), while alheroselerotc strokes fe pale infarctions, ‘a hemorthagic: 61 ‘Note: This material is copyrighted. All rights reserved. (Edward F. Goljan, M.D.) 2002 11, 62-year-old man complains of weakness in his left arm and tremor in his hands that prevents him from writing legibly. He has a shuffling gat when he enters the examining room. His deep tendon. ‘reflexes are normal. The patients clinical findings most closely relate to. ‘A. amyotrophic lateral sclerosis B. atherosoleroti stroke pure motor stroke ‘D, Parkinson's disease BE. _post-poliomyelitis syndrome ‘Answer: D~ this i due to deficieney of dopamine in substantia nigra neurons 12, A 30-year-old woman states that she bums her hands without feeling any pain. Physical exam reveals decreased pain and temperature sensation in the upper extremities, atrophy of the intrinsic ‘muscles of his hand, and abnormal deep tendon teflexes in the upper extremity. The pafient most Tiely has A. syringomyelia B. multiple sclerosis C___ amyotrophic lateral sclerosis D, spinal cord tumor E Badeficiency Answer: AALS has:no sensory changes, while syringomyclis does slong with motor changes. 1B. A febrile 28-year-old man with AIDS and a CDT helper eount of $0 cellsful. develops focal ‘epileptic seizures. A CT soen reveals multiple ring enbancing lesions inthe brain. The pathogenesis. ofthis patient's CNS disorder is most closely related to... FOO BE Answer: C- toxoplasmosis is the MCC of a space occupying lesion inthe brain in AIDS, Note the low hhelper cell count, Toxo is found in cat iter and in undercooked meat. 14, Which of the following is moce often associated with Escherichia coli than Pseudomonas aeruginosa? Osteomyelitis from puncture wounds through rubber footwear “Malignant otitis externa ina diaheie patient Pneumonia in an intensive care unit Death in patient in a burn wnt Meningitis ina newborn ‘Answer: B- it is the second MCC of neonatal meningitidsepsis, with group B streptococcus (S. ‘agalactiad) the MC. All the ther disorders characterize Pseudomonas 15, A.55-year-old man has broad-based ataxia and loss of pain and temperature sensation in the lower ‘extremities. Both pupils accommodate but do not constrict with direct light stimulation. You would ‘expect the spinal uid to exhibit... RooEP A. encapsulated yeast with narrow-based buds B. _ancutrophil dominant cell count © apositive VDRI D. spirochetes E. _ xanthochromia Answer: C-the patient has tabes dorsalis (neurosyphilis). Note the Argyil-Robertson popil. 16. A biopsy finding in the cercbellum ftom a comatose 28-yr-old man revealed Purkinje cells with an eosinophilic inclusion. The patient worked in a wild animal park The vector most likely responsible for this patent's disease isa A, dog a ‘Note: This material is copyrighted, All rights reserved, (Edward F. Golja, MD) 2002 sensorineural hearing loss, This patient most likely bas... A. tuberous selrasis B, the dysplastic nevus syndrome C. acanthosis nigricans D. multiple seborsheic keratoses B, _neurofibromatosis -Auiswer: F=the patent has neurofibromatosis with newreflbromas snd café wu lit spots, 418 A spinal tap in a 28-yrold mun with AIDS has numerous yeast forms with a nerrow based bud. ‘You would expect which ofthe following groups of spinal Auid results in this patient? Note: the ‘square norms values CSF glucose and protein Re CSE protein a Ae CSF glucose Answer: B- the patient has eryptocoocus. CSF protein is elevated and lucose is low. 19, An autopsy performed on a 29-yr0ld man teveals 4 necrotic frontal lobe absoeas, Histologic exam reveals numerous wide-sngled, non-septate hyphae. Based on the morphology ofthe patho, the patient most likely was... ‘analeoholic cave explorer missionary is Africa an insulin dependent diabetic ‘pig firmer who ate raw bacon, Answer: D- the patient fas rhinooerebral mucormycosis 20. A 23-ye-o san, who died in a car secident, hada large clot in the epidural space overlying the lft hemisphere. Which of the following isthe most common mechanism responsible forthe wutopsy ‘finding in this patient? A. Cerebral ischemia with neuronal degeneration B. Rupture ofa congenital aneuryemn ©. Stall facture with an arterial bleed ‘D. — Skull fracture with a venous bleed BE ‘Embolism fim the left heart Answer: C~ the patent had an epidural hematoma 21. An elderly patient in a nursery home fell and hit her head. She died 3 days later in the hospital. At autopsy, 4 large blood clot overlying the right hemisphere was noted between the dura and ‘arachnoid membranes, Which of the following best explains the aulopsy finding in this patient? ‘A. Cerebral ischemia with neuronal degeneration B. Rupture of a congenital snearysen C. Stal fracture with an arterial bleed D. Stall facture with a venous bleed! 6 ‘Note: This material is copyrighted. AM rights reserved. (Edward P. Goljan, MD.) 2002 E, _Embolism from the left heart Answer: D- the patient had a subdural hematoma with rupture of the bridging veins between the arachnoid and dura 22. A S8-yr-old man with severe diabetes mellitus died from a stroke. At autopsy, a large blood clot ‘was noted in the area ofthe putamen and globus pallidus. Which of the following best explains the ‘mechanism for the autopsy finding in this pation? A. Cexebral ischemia with neuronal degeneration. B. Rupture of congenital aneurysm C.__Asteriovenous malformation 1D. Embolism from the left heart B, _ Pootly controlled hypertension Answer: E~the patient had a hypertensive bleed. 23. A 58-yr-old man with adult polycystic kidney ied, At autopey, the surface of the brain was covered by blood. Which of the following isthe most offen responsible for initiating the autopsy finding inthis patient? | ‘Neoplastic process involving astrocytes ‘Answer: A- subarachnoid hemocthage from rupture of congenital berry aneurysm 24, - An autopsy finding ftom a 35-yr-old man with severe frontal headaches revealed a hemorrhagic and ‘neerotic mass that traversed the corpus csllosum and involved both sides ofthe brain. Which ofthe ‘ollowing best describes the pathogenesis of this patient's bran lesion? A. Benign neoplasm decived from oligodendrocytes 1B. Malignant neoplasm derived from astrocytes ‘C. _Infeacercbral hematoma due to hypertension D. — Embolism from the left heart EB. Atherosclerotic stroke Answer: B~ the patient has a glioblastoma multifoeme, the MC primary brain tumor in adults. 25, An incidental finding noted at an performed on a 45-y-old woman was a bard, popcorn shaped lesion attached to the dura. Tt impinged on the verebral eortex but did not invade brain tissue. Which of the following best deseribes the derivation ofthis brain lesion? AL Astrocytes B. Oligodendrocytes i elle D. ial celle E_ Arachnoid granulations Answer: E- the patient had a meningioma, which isthe most common benign primary brain tumor in adults It is a common cause of focal new onset seizures. Psammoma bodies are present in the ‘tumor. 26. Unilateral papilledema would most likely be associated with which one of the following clinical conditions? A. Glaucome B. Uncal herniation C. Multiple sclerosis D. Aleheimers disease B. Hyperatremia Answer: B~ papilledema implies sn inerease in intracranial pressure 64 ‘ote: This material is copyrighted. AI rights reserved. (Edvaed F. Goljan, M.D.) 2002 27, Which of the following clinical seenatios correlates best with the rapid inorease in primary central ‘nervous system malignant lymphomas in the United States? A. Increase in Epstein-Basr vieus infections B. Increase in human immunodeficiency virus infections C. _Tnarease in prion-elated central nervous system disease 1D. Increase in slow virus diseases in irsmanodeficient patients E. _Inetease in radiation therapy of metasatio cancers to the bexin Answer: B- the virus itself is responsible forthe cancer. 28. One week after an upper respiratory infection, a 25-year-old man develope weakness in the lower legs that is progressing into his upper torso. A spinal tap report indicates increased protein, a ‘normal glucose, a negative gram stain, and 20~30 lymphocytes mononuclear cells. Which of the following categories of disease best explains the clinical and laboretory findings in this patient?” A. Inbom error of metabolism B Atherosclerotic disease C.Demyelinating disease D. Neoplastic disease B, _ Infectious disease Answer: C- the patent has Guillain-Barre disease, an autoimmune disease with destruction of myelin in ‘the peripheral nervous system, 29, A 32-year-old woman complains of recurent pisodes af vinging inthe right ear along with ¥ sense of fullness behind the ear drum, She vccesiontlly experiences bouts of dizeinest where "the room seems to spin around”. On physical exam, the Weber test lateralizes to the left ear and the Rinne ‘test demonstrates that air conduction is longer than. bone conduetion in both ears. The tympanic ‘membrane has a normal fight reflex. Horizontal nystagmus is present in the right eye, Which ofthe following best explains the pathogenesis ofthis patent's disorder? ‘A. Right midalle ear infection B. Tumor of the acoustic nerve ©. Increased endolymph in the inner ear D. Brain tumor in the cerebellopoatine engle E. _ Demyelinating disease involving the acoustic nerve Amsver: C~ the paticat has Meniere's disease with sensorineural hearing loss 30, A ow woeks ago, a 65-year-old man presented with a painless unilateral loss of vision described as a “ourtsin suddenly going down and then coming up." He now presents with a sudden onset of ‘expressive aphasia, and contralateral lemiparesis and sensory loss, CT seans of the brain on. admission and after 24 hours reveal no evidence of hemorebaye. Which ofthe following pathologic _Provesoes best expleins the pathogenesis of this patient's clinical findings inthe past and present? ‘A. Embolism B. Atherosclerosis C.Lacunar infarction D. Intracerebral bleed Subarachnoid hemorrhage ‘Answer: B~ the patient had an atheroscleroti stroke preceded hy amaurosis fugex, which is embolization of atheromatous debris ftom plaque most commonly located atthe bifurcation of the carota artery 31, A.45-yr-old wife of a pig farmer develops focal new onset epileptic seizures. Her husband has a habit of eating raw bacon. An MRI of her brain reveals multiple calefied eysts. Which of the following applies to the pg, the husband, and the patient regarding the pathogen responsible for her disease? Pig Husband Wite (patient) ‘A Definitive host Intermediate host Definitive host, B. Intermediate host Intermediate host C._ Definitive host Intermediate host Note: This material is copyrighted. All rights reserved. (Edward F. Goljan, M.D.) 2002 D. Intermediate host Definitive host Definitive host B, Intermediate host ‘Intermediate host Intermediate host ‘Answer: 1 the patient has cystcercosi. The patient's husband must have eaten a underoookee pork with larva of Taenia solium in the meat Gntermediate host). Adults that Ind eggs developed in the firmer (definitive host). The eggs were transferred to the wile and they developed! into larvae (cysticersi wife isthe intermediate host) inthe GI tract which disseminated tothe brain, 32, A normotensive 29-year-old man with AIDS is beginning to have significant visual loss im both ‘eyes. His CD, T helper count is 50 cells. Retinal exam exhibits eotton wool exudates in both ‘eyes. The infectious agent that is responsible for this patient's central nervous system disorder Delongs to which of the following subtypes of microbial pathogens? A. Virus BL Fongus: C. Bacteria D. Helminth B. _ Sporozoan “Answer: A~ the patient has CMV relat, It should be teated with ganciclovir IF that does not work, oseamet should be tried. 33, A 28-year-old man, who lives in the Northeast, presents with x sudden onset of drooping and Arooling out of bath sides of his mouth, inability to close both eyes, and slurred speech. His wife states that a few weeks ago he removed a tick from his right thigh. In afew days following removal ofthe tick, & concentric, erythematous rash developed in that same area, The ifeotious agent tat is responsible for this patients previous skin disorder and current neurological findings belongs to which of the following, subtypes of microbial pathogens? A. Views B. Fungus © Ricketts D. Protozoan B. _Spirochete “Answer: E-the patient has Lyme's disease with bilateral Bell's palsy. Borrelia bungdorfi isa spirochete ttanumitted by an Ixodes tick. The skin lesion was erythema chronicum migrans 34. A 2-month-old infant has a progressive incrée in head circumference. The pediatrician suspects hydrocephalus and onders an MRL The MRI reveals dilatation of the third ventricle and lateral ventricles. Which of the following i the most likely nus ofthe patient's condition’? A Obstruction ofthe arachnoid granlations B. Obstruction ofthe aqueduct of Sylvius ©. Dandy-Walker syndrome D. Amold-Chiari syndrome BE. Tuberous sclerosis Answer: B- this is an obstructive hydrocephalus. 35, A 45-yr-old man notices weakening in his ability to open jars with his vight hand, He also notes “itching” in the muscles of his legs and back and an inequality in the size of his-calf and thigh ‘muscles, withthe muscles on the left much smaller than those on the right. Physical exam reveals trophy of te intrinsic museles ofboth hands, the right eater than the left and absent deep tendon reflexes inthe upper extremity and left leg. Muscle faciculations are noted inthe right forearm, lft calf, and left thigh. The Babinski sign is negative. Seusory function is intact. Which of the following pathologic processes best explains the pathogenesis of the patients neurological findings? ‘A, Triplet repeat disorder 1B. Demyelinating disease C._Degeneration of anterior born cells, 1D. Metaboli disorder involving copper 66 ‘Note: This material is copyrighted, All rights reserved, (Edward F. Goljan, M.D.) 2002 E. _ Degenerative changes inthe cervical spinal cord Auswer: C- this is classic amyotrophie lateral sclerosis (ALS) oa

Вам также может понравиться